HEADACHE CASES

1. A 24-year old nurse calls your office. She has had four episodes of an excruciating right-sided headache in the last six months. Twice there was an associated tingling of the left side of the face and left hand. The pain, which is throbbing, builds up over an hour or two until she is forced to lie down in a dark room. She experiences nausea and vomiting, and blurred vision. The headache usually subsides once she is able to sleep. She also gets less severe headaches, also on the right, relieved by 2-4 aspirin per day. She denies family history of headache. Her sister works on a neurology ward and insisted she call for an appointment.

2. An 18-year-old college student comes to the ER with a headache which is holocephalic, steady, and unaccompanied by any other symptoms or findings. You give him a prescription of Fiorinal, (butalbital, aspirin and caffeine). He returns on a different shift 2 days later with continuing symptoms. The Fiorinal was slightly effective, bul,he thinks he took too much. He is slightly groggy, but there are no other findings. He has no history of drug use or chronic headaches. The physician prescribes a shot of Demerol hoping he can stop the headache so the student can study for his finals. When he returns 12 hours later, there is little change in his story or exam. He demands another shot.

3. A 68-year-old woman is referred to you by her dentist. She has suffered some tooth decay because she refuses to brush her teeth due to facial pain. She explains that she has developed an excruciating pain in the right side of her face. It is exacerbated by facial movements or touch, and is most severe in the morning. She is suing the dentist as she claims it all started after a root canal.

4. A 35-year-old businessman has a six-year history of twice weekly headaches which have increased to almost daily. They are usually non-throbbing and occur towards the end of the day. He has tried many off-the-shelf medications, but finds himself taking a handful of pills without much relief. His friend gave him a prescription drug that worked. He would like for you to give him some. There are no other symptoms or neurologic findings.

5. A woman has a new onset seizure. She is thoroughly examined including a CT, LP, and EEG. No findings were abnormal. Two days later, she calls the office with complaints of a headache that developed the day before. She has not been able to get out of bed due to the pain.

6. A 40-year-old policeman states that he has headaches which are unilateral, occur once or twice per night, but at least once around 3 AM. They last only an hour, but they are excruciating and he paces the floor. He claims his eye swells and tears. This has been going on for a week. His mother had migraines, but nothing like this.

7. A 28 year old white female complains of headaches for 1 year, recently daily. They are often throbbing, usually bitemporal, and do not usually cause too much nausea, although she has vomited once or twice. She also says her vision has changed, but she went to get her glasses checked and they told her they were fine. Other pertinent history is obtained that she had a child 9 months ago, and gained 80 pounds d-uring pregnancy. She has lost 30. On physical exam she is obese. Vital signs are normal. Fundoscopic exam shows bilateral disc margin blurring with a flame hemorrage in the right. Pupils are equally reactive. Visual fields are full on finger confrontation. There is a question of mild lateral rectus weakness on the right. The rest of the cranial nerves are normal, as is her strength, sensation, and reflexes. There is no Babinski. Coordination and gait are intact.

CT of the head is normal. EEG is normal. CSF is normal except for an opening pressure of 410 mm H20.

What is the name of this syndrome? And What severe disability is she at risk for?

What would you expect to see on formal visual field testing (perimetry)?

HEADACHE CASE ANSWERS

1. Migraine with aura is likely. The side-locked location is a little worrisome for possible referred pain due to intracranial pathology. Remember that according to International Headache Criteria for migraine headache the patient has to have at least five headaches which are similar in nature. She is also overusing analgesics to a small extent, which could lead to medication-induced chronic headache. She needs an exam to rule out focal neurologic signs and more specific migraine therapy.

2. This person could be drug seeking. Signs which may point to this are a family history of drug abuse or alcoholism. However, he could also have his first prolonged migraine. This would be unusual, but not unheard of even though it is not unilateral or throbbing. On the other hand, you are obligated at this point to work him up because he is presenting with "the First and the Worst" headache of his life. He should have a blood screen for infection and metabolic disease, a CT scan without contrast to look for blood, and an LP to look for blood and infection.

3. She has trigeminal neuralgia. It will likely respond to anticonvulsants such as Neurontin or Tegretol. If she has no other signs of cranial nerve dysfunction she doesn't need imaging, though it is not unusual to do it as a precaution. Remember that TN is not associated with true sensory loss, although the patients will say they have a relative numbness in the area.

4. This is the more classic "rebound" or analgesic induced chronic tension-type headache. He should change his lifestyle somewhat, participate in exercise and stress reduction activities, stop the daily butalbital and perhaps get on a more stabilizing preventive such as amitriptyline.

5. The two most important clues to the correct diagnosis is that she recently had an LP, and when she tries to get up from bed the headache worsens. This is acute post-LP, or "hypoliquorric" headache. It is treated with strict flat bedrest for three days; if they are not better at the end of that time an autologous blood patch to the epidural lumbar area by an anesthesiologist is almost always curative. IV or high dose PO caffeine can sometimes help, but I can't think of anything more uncomfortable than to be buzzing but have to stay in bed, so I don't usually suggest it.

6. These are cluster headaches, the most painful headache there is most likely. Acutely a triptan intranasally or 100% oxygen can abort it; preventively the most helpful are valproate, verapamil, lithium and/or pulse steroids. It will usually go away on its own in 6-8 weeks.

7. Idiopathic intracranial hypertension. She is at risk for visual loss, often not noticed by the patient until it is severe, due to fovial sparing. On perimetry testing she will have an enlarged blind spot and sometimes a "nasal step" of visual loss. If acetazolamide does not decrease her intracranial pressure, surgery to open a window in the optic nerve sheath ("optic nerve sheath fenestration") is indicated to preserve her eyesight.

 

1) A 65 year old female Kamala gives the history of severe unilateral headache on the right side and complains of blindness since 2 days .On examination there is a thick cord like structure on the lateral side of the head. The ESR is 80 mm/Hr in the first hour. The most likely diagnosis is

a) Temporal arteritis

b) Migraine

c) cluster headache

d) sinusitis

2) A 60 year old man complaining of chest pain since 6 hrs is diagnosed as acute MI . Angiography showed the involvement of anterior descending branch of left coronary artery.

The most probable site of involvement is

a) Anterolateral

b) Posterior

c) Inferior

d) Septal

3) Pulmonary Kerley B lines are seen in all the following except

a) Pulmonary edema

b) Mitral valve disease

c) Interstitial fibrosis

d) Broncho alveolar Ca

4) Infective endocarditis is least likely to occur in

a) ASD
b) Small VSD

c) MVP

d) TOF

5)In a patient with chronic AF with a regular beat of 60/min, the most probable cause is

a) sleep

b) digitalis toxicity

c) Sino nodal block

d) Hypothyroidism

6)An HIV positive female has an indurated ulcer over the tongue. Laboratory findings show growth in cornmeal agar at 20 Degrees, microscopy showing hyphae and growth in human serum at 37 degrees show budding yeasts. The probable cause is

a)Candida albicans

b)Histoplasmosis

c)Blastomycosis

d) Coccidiodomycosis

7) Kallu, a 30 year old man, presented with subcutaneous itchy nodules over the left iliac crest. On examination, they are firm, non tender and mobile .Skin snips contain microfilaria and adult worms of

a) Loa Loa

b) Onchocerca volvulus

c) Brugia malayi

d) Mansonella perstans

8)In an ICU patient on invasive monitoring, all are causes of bacterial sepsis except

a)Orotracheal intubation

b)Intra arterial line

c)Humidified air

d)Central venous catheter

9) In Polycythemia vera, all the following are seen except

a) thrombocytopenia

b) increased GI bleed

c) thrombosis

d) Transient visual loss

10) A stem cell disorder affecting all the 3 cell lines - platelets, RBC's and leucocytes is

a) Hemolytic anemia

b) Paroxysmal cold haemoglobinuria

c) PNH

d) Blackfan Diamond syndrome

11) In beta thalassemia, there is

a) increase in Beta chain , decrease in Alpha chain

b) decrease in Beta chain, increase Alpha chain

c) decrease in Beta chain, decrease Alpha chain

d) Increase in Beta chain, increase Alpha chain

12) Non caseating granulomas are seen in all the following except

a) Tuberculosis

b) Byssinosis

c) Hodgkin's Lymphoma

d) Metastatic carcinoma of lung

13) All are paraneoplastic syndromes except

a) cerebellar degeneration

b) Progressive Multifocal leukoencephalopathy

c) Amyotropic lateral sclerosis

d) Opsoclonus myoclonus

14) In Burkitts lymphoma, translocation seen is

a) 12-14 translocation

b) 8-14 translocation

c)

d)

15) Ingestion of arsenic causes

a) Hepatic Ca

b) Hepatic adenoma

c) Noncirrhotic portal fibrosis

d) Hepatic cirrhosis

16) In malignant hypertension, hyperplastic angiitis is seen in all except

a) Peripancreatic fat

b) Kidney

c) Heart

d) Periadrenal fat

 

17) In a chronic smoker, a highly malignant, aggressive and metastatic lung carcinoma is

a) Squamous cell Carcinoma

b) Small cell Carcinoma

c) Adenocarcinoma

d) Large cell Carcinoma

 

18) All are true about Neisseria gonorrhea except

a) Gram positive cocci

b) causes stricture urethra

c) Involves seminal vesicles and spreads to epididymis

d) drug of choice is Ceftriaxone

19) A 30 year old male, Kallu, with a history of sexual exposure comes with a painless indurated ulcer over the penis with everted margins. The diagnosis is

a) Syphilis

b) Chancroid

c) LGV

d) Granuloma inguinale

20) All are features of peripheral neuritis in a patient with Hansens Disease except

a) Predominant sensory involvement

b) Decreased tendon reflexes

c) Mutilations

d)

21) A 32 year old male, Kallu, who recently visited a sea coast presented with an ulcer over the left leg. The probable cause is

a) Pasturella multocida

b) Micrococcus halophilus

c) Vibrio vulnificus

d)

22) An 8 year old boy, Kallu, is brought to the casualty with a history of consuming something while playing outside in a play ground. On examination, there was altered sensorium, hyperpyrexia, dilated pupils. Most probable cause is

a) Dhatura poisoning

b) Organophosphorus poisoning

c) Parthenium poisoning

d) Barbiturate poisoning

23) All the following viruses cause stomach and colon cancer except

a) H pylori

b) EB virus

c) HBV

d) HIV

24) Which of the following is pathognomonic of renal disease

a) Hyaline casts

b) coarse granular casts

c) cystine oxalate crystals

d) epithelial cells

25) Hypoglycemia is seen in

a) Acromegaly

b) Cushings syndrome

c) Hypothyroidism

d) Hypopitutarism

26) A 76 year old male comes with a history of frequent falls and difficulty in looking downwards and laterally. The diagnosis is

a) Alzheimer's disease

b) Supranuclear palsy

c) Amyotropic lateral sclerosis

d) Oculomotor nerve palsy

27) The commonest side effect of cisplatin in a patient using it for esophageal carcinoma is

a) ATN

b) Thrombocytopenia

c) Hepatic failure

d) Cardiomyopathy

28) The most common late CNS complication of HIV is

a) Dementia

b) Ataxia

c) Seizures

d) Delirium

29) In a patient with acute liver failure, the best prognostic indicator is

a) Serum albumin

b) Serum AFP

c) Serum bilirubin

d) Factor V estimation

30) The commonest hepatotropic virus causing increased chronic carrier state is

a) HEV

b) HAV

c) HBV

d) HCV

31) Regarding Addisonian pigmentation, all are true except

a) involves moles and scars

b) involves palmar creases

c) does not involve oral mucosa

d) involves sole pads

32) In an HIV positive patient with tuberculosis, all are true except

a) decreased cavitation

b) increased sputum positivity

c) highly variable tuberculin test

d) decreased fibrosis

33) In essential hypertension, changes seen in the heart are

a) cardiac cell hyperplasia

b) cardiac cell hypertrophy

c) increase in the mitochondrial number

d)

34) In a 32 year old male presenting with the following blood chemistry : Na+ 135, K+ 5.0, HCO3- 14.0, Cl- 116, PO4 5.0, SO4 5.0, Mg 2.0, Ca 8.0, the anion gap is

a) 10

b) 15

c) 13

d) 20

35) Pyramidal tract involvement with absent ankle jerk is seen in

a) Frederick's ataxia

b) Subacute combined degeneration of the spinal cord

c) Lathyrism

d) Tabes dorsalis

36) A patient with BHP underwent TURP and developed altered sensorium. The cause is

a) Hyponatremia

b) Hypernatremia

c) Hypomagnesemia

d) Hypokalemia

37) A patient of BHP underwent TURP under bupivacaine spinal anesthesia. One hour later, he developed nausea, vomiting and altered sensorium. The cause possibly is

a) Hypernatremia

b) Water intoxication

c) Bupivacaine overdose

d) Rupture Bladder

38) Neuroblastoma differs from Wilm's tumor radiologically by all except

a) same location

b) intraspinal extension

c) calcification

d) aorta and IVC are not eroded but pushed aside

39) A 60 year old male, Kallu, on CT scan was found to have a solid tumor involving the inferior vena cava and renal vein. The tumor was within the Gerota's fascia. All the following can be done except

a) chest X-ray to rule out pulmonary metastasis

b) pre-operative radiotherapy is not favorable

c) IVC invasion is inoperable

d) pre-operative biopsy is not indicated

40) A male patient, Kamal, aged 50 years presents with gradually progressive hard scrotal swelling on the right side. All the following can be done except

a) inguinal exploration

b) chest Xray to rule out pulmonary metastasis

c) CT abdomen

d) biopsy through the scrotal skin

41) Orhcidectomy is done in all the following except

a) male breast cancer

b) filarial epididymo-orchitis

c) seminoma testes

d) prostate cancer

42) The treatment of choice in congenital hydrocele is

a) Eversion of the sac

b) Excision of the sac

c) Herniotomy

d) Lord's procedure

43) In a patient with pheochromocytoma, all the following are seen except

a) diarrhea

b) orthostatic hypotension

c) episodic hypertension

d) weight gain

44) Splenectomy is done in all the following except

a) splenic abscess

b) hereditary spherocytosis

c) sickle cell anemia

d) myelofibrosis

45) A patient with recurrent attacks of cholelithiasis had, on ultrasound examination , a dilated CBD of 1 cm. The next best line in management is

a) ERCP

b) PTC

c) intravenous cholangiogram

d) cholecystostomy

46) A 40 year old male, Kallu, a chronic alcoholic, diagnosed as cirrhosis presents with a lump in the right lobe of the liver. Serum AFP levels are not elevated. The diagnosis is

a) fibrohyperplasia

b) hepatocellular carcinoma

c) hepatocellular adenoma

d) secondary from carcinoma colon

47) A 60 year old male diagnosed to have carcinoma stomach had, on CT scan of the abdomen, a mass measuring 4x4 cm in the antrum with involvement of celiac nodes and right gastric nodes. The management of choice is

a) palliative

b) subtotal gastrectomy

c) total gastrectomy

d) chemotherapy and radiotherapy

48) A 70 year old male, Kamal, presents with lower GI bleed for the last 6 months. On sigmoidoscopic examination, there is a hard non-obstructing mass of 4 cms about 3 cms above the anal verge. The treatment of choice is

a) anterior resection

b) abdominoperineal resection

c) defunctioning anastomosis

d) colostomy

49) A 14 year old girl, Salu, a regular swimmer presents with sudden onset of pain abdomen, abdominal distension and fever of 390, but without obliteration of the liver dullness. The diagnosis is

a) ruptured typhoid ulcer

b) ruptured ectopic pregnancy

c) primary bacterial peritonitis

d) UTI with PID

50) Left sided pulmonary hypertension is best treated by

a) splenectomy

b) spleno-renal shunt

c) portocaval shunt

d)

51) A 28 year old male, Mallu, a chronic alcoholic, presents with sudden onset of epigastric pain that is radiating to the back. All the following can be seen except

a) hypocalcemia

b) increased serum amylase

c) low serum lipase

d) increased LDH

52) A 28 year old male patient is brought to the casualty with severe hypotension following a car accident. On examination no external bleed or injuries are seen. The cause of hypotension is possibly

a) intrathoracic and abdominal bleed

b) fracture rib

c) intracranial bleed

d) neurogenic shock

53) On prenatal ultrasound examination, the diagnostic feature of congenital diaphragmatic hernia is

a) absence of gas bubble under the diaphragm

b) mediastinal shift with normal heart axis

c) peristalsis in the thoracic cavity

d) a cyst behind the left atrium

54) A 40 year old male, Kallu, a chronic smoker, presents with claudication and a medial leg ulcer. For the past one month, he gives a history of rest pain. All the following can be used to relieve the rest pain except

a) Omentoplexy

b) conservative amputation

c) lumbar sympathectomy

d) femoropopliteal bypass

55) A male patient aged 30 years following a road traffic accident presents with fracture of 4th to 10th ribs and respiratory distress. He is diagnosed to have flail chest and PaO2 is < 60%. The treatment of choice is

a) fixation of ribs

b) strapping of the chest

c) IPPV with oral intubation

d) tracheostomy

56) A post-operative patient presents with duodenal leak and peritonitis with massive contamination. The appropriate management of choice is

a) duodenostomy + feeding jejunostomy + peritoneal lavage

b) total parenteral nutrition

c) duodenojejunostomy

d) four quadrant peritoneal lavage

57) A 52 year old male executive is seen in the casualty with hypotension, vomiting bright red blood at home and in the hospital. There is no previous suggestive history. The estimated blood loss is around 2 liters. The diagnosis is

a) esophageal varices

b) gastritis

c) duodenal ulcer

d) Mallory-Weiss tear

58) All are true about the right kidney except

a) it is related to the duodenum

b) it is lower than the left kidney

c) the right renal vein is shorter than the left

d) right kidney is preferred over the left for transplantation

59) Grade IV esophageal varices, on barium swallow appears as

a) a thick band

b) mucosal folds above the carina

c) mucosal folds below the carina

d)

60) A female patient presents with pigmentation of the lips and oral mucosa and colonic polyps. Her sister also has the same history. The diagnosis is

a) Peutz-Jeghers' syndrome

b) Carcinoid

c) melanoma

d) villous adenoma

61) The commonest site of carcinoma prostate is

a) Transitional zone

b) Central zone

c) Peripheral zone

d) Anterior zone

62) Dissociative sensory loss is seen with

a) Tabes dorsalis

b) Syringomyelia

c)

d)

63) In a patient with raised IgA levels, the commonest finding is

a) proteinuria

b) GI bleed

c) hematuria

d) hypertension

64) In a patient with choledochal cyst, bile diversion into the small intestine is contraindicated because of risk of

a) malignancy

b) recurrent cholangitis

c) pancreatitis

d) increased gall stones

65) Thyrotoxicosis differs from malignant hyperthermia by

a) muscle rigidity

b) CPK

c) hypothermia

d)

66) In a female with sudden onset of severe headache, on CT, a diagnosis of subarachnoid hemorrhage was made. The most common site of subarachnoid hemorrhage is

a) subdural venous sinuses

b) middle meningeal artery

c) Berry aneurysm rupture

d) basilar artery

67) Chamavati, a 30 year old female from rural Assam with a history of chronic tobacco chewing since 15 years of age, presents with difficulty in opening the mouth. On oral examination, no ulcer is seen. The diagnosis is

a) non-ulcerative carcinoma of buccal mucosa

b) submucous oral fibrosis

c) Temporomandibular joint arthritis

d)

68) A 40 year old female patient, Chamavati, presents with a lump that is gradually increasing in the parotid region. On oral examination, the tonsil was pushed medially. On biopsy, pleomorphic adenoma was diagnosed. The appropriate treatment is

a) Superficial parotidectomy

b) Lumpectomy

c) Enucleation

d) Conservative total dissection

69) A female patient, 45 years of age, with a family history of breast carcinoma showed diffuse microcalcification on mammography. On biopsy, intraductal carcinoma in situ was found. The appropriate management is

a) simple mastectomy

b) quadrantectomy

c) radical mastectomy with axillary sampling

d) chemotherapy with CMF regime

70) Tram track appearance on CT scan of the head is seen in

a) Sturge Weber syndrome

b) von Hippel Lindau disease

c) Tuberous sclerosis

d) neurofibroma

71) A patient presents with minimal pleural effusion on the right side. The best method to detect this would be

a) right side chest Xray

b) Left sided chest Xray

c) Left lateral decubitus chest Xray

d) Right lateral decubitus chest Xray

72) A 40 year old farmer with a history of recurrent attacks of porphyria complains of itching when exposed to the sun and maculopapular rash on sun-exposed areas. His symptoms are exaggerated in the summer. The diagnosis is

a) Seborrheic dermatitis

b) Contact dermatitis

c) Psoriasis

d) Porphyria cutanea tarda

73) An 8 year old boy presents with a well defined annular lesion over the buttock with central scarring that is gradually progressing over the last 8 months. The diagnosis is

a) annular psoriasis

b) lupus vulgaris

c) tinea corporis

d)

74) In adult polycystic kidney, all are true except

a) Cysts are seen in the liver, spleen and the pancreas

b) Hematuria occurs

c) Hypertension is rare

d) Autosomal dominant transmission is seen

75) In thymoma, all are seen except

a) red cell aplasia

b) hyperalbuminemia

c) hypogammaglobulinemia

d) myasthenia gravis

76) An adult presents with oval scaly hypopigmented macules over the chest and the back. The diagnosis is

a) Leprosy

b) Lupus vulgaris

c) Pityriasis versicolor

d) Lichen planus

77) The characteristic nail finding in lichen planus is

a) pitting

b) pterygium

c) Beau's lines

d) Hyperpigmentation of the nails

78) In an 8 day old child with no history of consanguinity in the parents, the mother reports blisters and peeling off of the skin at the site of handling and pressure. There was a similar history in the previous child which proved to be fatal. The diagnosis is

a) Bullous pemphigoid

b) Congenital syphilis

c) Congenital epidermolysis bullosa

d) Letterer-Siwe disease

79) The most common complication of hypermature sclerotic cataract is

a) dislocation of the lens

b) phakomorphic glaucoma

c) uveitis

d)

80) A 14 year old boy complains of pain during reading . On examination, his both eyes are normal and vision with non Snellen's reading is 6/5. He still complains of pain on occluding one eye. The diagnosis is

a) myopia

b) pseudomyopia

c) hyperopia

d) emmetropia

81) A 16 year old boy complains of pain in the right eye. After refractometry , he was prescribed a + 3.5 D sphere lens. The cover test is normal. There is no heterophoria. The diagnosis is

a) organic amblyopia

b) anisometric amblyopia

c) emmetropic amblyopia

d) toxic amblyopia

82) A patient complains of pain in both eyes with congestion, blurring of vision, photophobia and mucopurulent discharge since one day. Many cases have been reported from the same community. The causative agent is probably

a) adenovirus

b) enterovirus 70

c) herpes simplex

d)

83) A male patient with a history of hypermature cataract presents with a 2 day history of ciliary congestion, photophobia, blurring of vision and on examination has a deep anterior chamber in the right eye. The left eye is normal. The diagnosis is

a) phakomorphic glaucoma

b) phakolytic glaucoma

c) phakotoxic glaucoma

d) phakoanaphylactic uveitis

84) A 60 year old male patient operated for cataract 6 months back now complains of floaters and sudden loss of vision. The diagnosis is

a) vitreous hemorrhage

b) retinal detachment

c) central retinal artery occlusion

d) cystoid macular edema

85) A 12 year old boy presents with recurrent attacks of conjunctivitis for the last 2 years with intense itching and ropy discharge. The diagnosis is

a) vernal conjunctivitis

b) phlyctenular conjunctivitis

c) trachoma

d) viral conjunctivitis

86) A 25 year old lady presents with severe sudden onset of pain, corneal congestion, photophobia and deep anterior chamber in the right eye. The left eye is normal. Xray pelvis shows sacroiliitis. The diagnosis is

a) anterior uveitis

b) posterior uveitis

c) intermediate uveitis

d) scleritis

87) A 30 day old neonate was presented with a history of photophobia and excessive lacrimation. On examination, both the lacrimal duct systems are normal, but there was a large cornea and corneal haziness. The diagnosis is

a) megalocornea

b) keratoconus

c) congenital glaucoma

d) Hunter's syndrome

88) In high spinal anesthesia, seen are

a) hypotension and bradycardia

b) hypotension and tachycardia

c) hypertension and bradycardia

d) hypertension and tachycardia

89) A patient selected for surgery who was induced with thiopentone i.v through one of the antecubital veins complains of severe pain of the whole hand. The next line of management is

a) give i.v propofol through the same needle

b) give i.v ketamine through the same needle

c) give i.v lignocaine through the same needle

d) leave it alone

90) A patient who was on aspirin for a long period was selected for an elective surgery. What should be done?

a) stop aspirin for 7 days

b) infusion of fresh frozen plasma

c) infusion of platelet concentrate

d) go ahead with the surgery maintaining adequate hemostasis

91) A patient in the ICU was on invasive monitoring with intraarterial cannulation through the right radial artery for the last 3 days. Later, he developed swelling and discoloration of the right hand. The next line of management is

a) stellate ganglion block

b) brachial block

c) radial nerve block on the same side

d) application of lignocaine jelly over the site

92) A 3 year old child with severe sensorineural deafness was prescribed hearing aids, but shows no improvement. The next line of management is

a) cochlear implant

b) fenestration surgery

c) stapes mobilization

d)

93) A 4 year old girl diagnosed as having multiple juvenile papillamotosis of the larynx presents to the casualty with mild respiratory distress. The next line of management is

a) tracheostomy

b) microlaryngoscopy

c) broad spectrum antibiotics

d) systemic steroids

94) A 28 year old female presents to the casualty with gradually increasing respiratory distress since 4 days. She gives history of hospitalization and mechanical ventilation with orotracheal intubation for 2 weeks. Now, she was diagnosed as having severe tracheal stenosis. The next line of management is

a) tracheal resection and end to end anastomosis

b) tracheal dilatation

c) systemic steroids

d) laser excision and stent insertion

95) A 30 year old male, following excision of the 3rd molar for dental caries presents with trismus, fever of 39.50 and swelling pushing the tonsil medially and spreading laterally posterior to the middle sternocleidomastoid. The diagnosis is

a) parapharyngeal abscess

b) retropharyngeal abscess

c) Ludwig's angina

d) submental abscess

96) A 20 year old male, following a road traffic accident was brought to the casualty. His right leg is shortened, internally rotated and adducted. The diagnosis is

a) fracture neck of femur

b) anterior dislocation of the hip

c) posterior dislocation of the hip

d) trochanteric fracture of the femur

97) An 8 year old boy presents with a gradually progressing swelling and pain since 6 months over the upper tibia. On Xray, there is a lytic lesion with sclerotic margins in the upper tibial metaphysis. The diagnosis is

a) Osteogenic sarcoma

b) Osteoclastoma

c) Brodie's abscess

d) Ewing's sarcoma

98) An 8 year old boy with a history of fall from 10 feet height complains of pain in the right ankle. Xrays taken at that time are normal without any fracture line. But after 2 years, he developed a calcaneovalgus deformity. The diagnosis is

a) undiagnosed malunited fracture

b) avascular necrosis talus

c) tibial epiphyseal injury

d)

99) A 6 year old boy has a history of recurrent dislocation of the right shoulder . On examination, the orthopedician puts the patient in the supine position and abducts his arm to 90 degrees with the bed as the fulcrum and then externally rotates it but the boy does not allow the test to be performed. The test done by the orthopedician is

a) apprehension test

b) sulcus test

c)

d)

100) Regarding bone remodelling, all are true except

a) osteoclastic activity at the compression site

b) osteoclastic activity at the tension site

c) osteoclastic activity and osteoblastic activity are both needed for bone remodelling in cortical and cancellous bones

d) osteoblasts transforms into osteocytes

101) A child is spinned around by his father by holding both hands. While doing this the child started crying and does not allow his father to touch his elbow.The diagnosis is

a) pulled elbow

b) radial head dislocation

c) Annular ligament tear

d) Fracture olecranon process

102)The structures piercing the clavipectoral fascia are all except:

a) Cephalic vein

b) Thoraco acromial artery

c) Lateral pectoral nerve

d) Lateral pectoral artery

103) Crossed oculomotor palsy is seen in occlusion of

a) Anterior cerebral artery

b) Middle cerebral artery

c) Posterior cerebral artery

d) Superior cerebral artery

104) Erection of penis is mediated by all the following except

a) Nervi erigentes

b) Pudendal nerve

c) Sacral plexus

d) Hypogastric plexus

 

105) Active transport of chemicals across the cell membrane is mediated by:

a) Channel protein

b) Carrier protein

c) G protein

d) Na+- K+ ATPase.

106) In a patient with respiratory rate of 14/min, tidal volume of 500 ml with a vital capacity 7000ml, the alveolar ventilation/minute is

a) 2000 ml

b) 4900ml

c) 7000ml

d) 7700ml.

107) During sigmoidoscopy, if the rectum is inflated with gas, increased peristalsis is seen in

a) Whole intestine

b) Distal colon

c) Proximal colon

d) Whole colon

108) In an unacclimatised person suddenly exposed to cold, the physiological effect seen is

a) Hypertension

b) Tachycardia

c) Shift of blood from shell to core

d) Non shivering thermogenesis

109) Spuriously high BP is noted in all the following except

a) Obesity

b) Thick calcified vessels

c) Small cuff

d) Auscultatory gap

110) Cerebellar herniation presents with all the following except

a) Loss of consciousness

b) Autonomic disturbances

c) Neck stiffness

d) Pupil dilatation

111) Positive feedback is seen in all the following except

a) LH Surge

b) Stimulation of the gastric secretion by histamine and gastrin

c) thrombolytic activity in the coagulation cascade

d) Entry of Ca into the sarcoplasmic reticulum

112)Vitamin K is needed for the post translational modification of

a) Carboxylation

b) Methylation

c) Hydroxylation

d) Transketolation

113) Amber codon refers to

a) Initiating codon

b) Mutant codon

c) Stop codon

d) Codon coding for multiple amino acids

114) At physiological pH, the most stable amino acid is

a) Histidine

b) Lysine

c) Arginine

d) Leucine

115) In cystinuria, amino acids excreted are all the following except:

a) Ornithine

b) Arginine

c) Lysine

d) Histidine

116) Dietary triglycerides are transported by

a) Chylomicrons

b) LDL

c) VLDL

d) HDL

117) In which of the following reaction, thiamine is not used

a) Alpha ketoglutarate to succinyl CoA

b) Glucose to pentose

c) Oxidative decarboxylation of Alpha keto amino acids

d) Lactate to pyruvate

118) In chromatography, mass movement of the substances is seen in

a) Electrophoresis

b) Diffusion

c) Osmosis

d) Paper chromatography

119) The type of chromatography in which proteins are bound to another substance is

a) Hydrophobic chromatography

b) Absorption ( Affinity ) chromatography

c)

d)

120) The end-product of citric acid cycle used in detoxification of ammonia in brain is

a) Oxaloacetate

b) Alpha keto glutarate

c) Succinate

d) Citrate

121) Right parietal lobe lesions cause all the following except

a) ideomotor apraxia

b) Self neglect

c)

d)

122) Investigation of choice for blood grouping in old blood stain on the cloth is

a) Precipitin test

b) Benzidine test

c) Acid dilution test

d) Hemin crystals

123) Interleukin secreted by the macrophages stimulating lymphocytes is

a) IL 1

b) INF Alpha

c) TNF Alpha

d) IL 6

124) Following injection of lymphokines, the same class of immunoglobulins are produced. This is referred to as

a) Clonal selection

b) Class switching

c)

d)

125) A patient receiving allopurinol requires dose reduction of

a) 6 mercapto purine

b) cyclophosphamide

c) Azathioprine

d) Cimetidine

126) An elderly hypertensive has diabetes mellitus and bilateral renal artery stenosis. The best management is

a) Enalapril

b) Verapamil

c) Beta blockers

d) Thiazides

127) A female suffering from psychosis, taking phenothiazines now complains of sudden onset of high grade fever, muscle rigidity and altered sensorium. The diagnosis is

a) Malignant hyperthermia

b) Neuroleptic malignant syndrome

c) Tardive dyskinesia

d) Akathesia

 

128) A patient on cisapride for Barrets ulcer suffers from pneumonia. The physician prescribes erythromycin. Which of the following is the correct statement

a) Increase the dose of cisapride needed

b) Increase the dose of erythromycin needed

c) Increased risk of Ventricular arrythmias present

d) Decreased bioavailability of cisapride

129) A 60 year old male comes to the casualty with acute retention of urine since 12 hours. On examination there was distended bladder . His son gives a history of taking some drugs by the patient since 2 days as he is suffering from depression . The most likely drug is

a) Amitryptiline

b) Chlorpromazine

c) Haloperidol

d)

130) Kallu, a 22 year old male suffers from decreased sleep, increased sexual activity , excitement and spending excessive money excessively for the past 8 days. The diagnosis is

a) Acute mania

b) Acute psychosis

c) Schizophrenia

d) Psychosexual disorder

131) Rathi, a 26 year old female diagnosed to be suffering from depression now for the past 2 days had suicidal tendencies, thoughts and ideas. The best treatment is

a) Amitryptiline

b) Selegiline

c) Haloperidol + Chlorpromazine

d) ECT

132) Kallu, a 22 year old single unmarried man is suffering from sudden onset of 3rd person hallucinations for the past 2 weeks. He is suspicious of his family members and had decreased sleep and appetite. The diagnosis is

a) Schizophrenia

b) Acute psychosis

c) Acute mania

d) Delirium

133) A 60 year old male suffering from auditory hallucinations is says that people staying upstairs are talking about him and conspiring against him. He dropped a police complaint against them but the allegations were proved to be wrong. The diagnosis is

a) Schizophrenia

b) Depression

c) Dementia

d) Delusional disorder

134) In India the commonest cause of juvenile onset of DM is

a) MODY

b) Fibrocalcific pancreatopathy

c) Gall stones

d) IDDM

135) A neonate has central cyanosis and short systolic murmur on the 2nd day of birth. The diagnosis is

a) TGV
b) TOF
c) VSD

d) ASD

136) An 8 year old female child following URTI developed maculopapular rash on the face spreading onto the trunk which cleared on the 3rd day without desquamation and tender post auricular and suboccipital lymphadenopathy. The diagnosis is

a) Measles

b) Rubella

c) Erythema infectiosum

d) Kawasaki disease(Mucocutaneous LN Syndrome)

137) A 4 month old HIV positive child following URTI ,developed sudden onset of breathlessness. The chest Xray shows hyperinflation. The O2 saturation was greater than 90%. The treatment of choice is

a) Nebulized acyclovir

b) i.v Ganciclovir

c) Ribavirin

d) Cotrimoxazole

138) All of the following are features of systemic juvenile Rheumatoid arthritis except

a) Rash

b) fever

c) Hepato Splenomegaly

d) Uveitis

139)A child climbs with alternate steps , builds a tower of 8-9 cubes , tells "I" but not his name and cannot say his age and sex. The probable age is

a) 24 Months

b) 36 Months

c) 30 Months

d) 48 Months

140) A child suffering from acute diarrhea is brought to the casualty and is diagnosed as having severe dehydration with pH of 7.23 , Serum Na -125 , Serum K- 3 , Hco3 16. The best IV fluid of choice is

a) 3% saline

b) Normal saline

c) N/3 saline + 10 % dextrose

d) N/3 saline + 5% dextrose

141) A child with recent onset of URTI after 2 days presents with acute onset of breathlessness, cough and fever. All of the following can be given except

a) Antibiotics

b) Antipyretics

c) O2 inhalation

d) Morphine

142)A 6 month old child having severe dehydration comes to the casualty with weak pulse and unrecordable BP. Repeated attempts in gaining IV access has failed. The next best step is

a) Venesection

b) Jugular vein catheterisation

c) Intraosseous IV Fluids

d) Try again

143) A 6 year old child with acute onset of fever of 104 F developed febrile seizures and was treated. To avoid future recurrence of seizure attacks what should be given

a) IV diazepam infusion over 12 Hrs

b) Paracetamol 400 mg + Phenobarbitone daily

c) paracetamol 400 mg 6th Hrly

d) Oral diazepam 6th Hrly

144 ) An 8 year old child suffering from recurrent attacks of polyuria since childhood presents to the paediatrics OPD. On examination, the child is short statured, vitals and BP are normal. Serum Cr 6 Mg %, HCO3 16 mEq , Na 134, K 4.2 On USG bilateral small kidneys

The most likely diagnosis is

a) Polycystic Kidney disease

b) Medullary cystic Kidney disease

c) Nephronophthisis

d) Reflux nephropathy

145) A 40 year old male presents with recurrent bouts of vomiting since 9 months because of

pyloric obstruction. The compensatory biochemical change is

a) Respiratory acidosis

b) Respiratory alkalosis

c) Metabolic acidosis

d) Paradoxical aciduria with hypo natremia and hypo chloremia

146)A diabetic female at 40 weeks of pregnancy delivered a baby by elective CS. Soon after birth the baby developed respiratory distress. The diagnosis is

a) Hyaline membrane disease

b) Transient tachypnea of the newborn

c) Congenital diaphragmatic hernia

d) Tracheo esophageal fistula

147)All are prognostic indicators of PIH except

a) Serum Uric acid

b) Low platelets

c) Serum Na

d) Elevated liver enzymes

148) In a woman on subdermal progesterone implant , the menstrual abnormality seen is

a) Amenorrhea

b) Menorrhagia

c) Metrorrhagia

d) Polymenorrhoea

149) Kalavathi ,a 29 year old nulliparous woman complains of severe menorrhagia and lower abdominal pain since 3 months. On examination, there is a 14 weeks size uterus with fundal fibroid . The treatment of choice is

a) Wait and watch

b) Myomectomy

c) GnRH analogues

d) Hysterectomy

 

150) A pregnant lady in the first trimester presented with random blood glucose of 177 mg /dl

The treatment is

a) Insulin

b) glipizide

c) Phenformin

d) Sulfonylurea

151) In a non-diabetic high risk pregnancy, the ideal time for NST monitoring is

a) 24 Hrs

b) 48 Hrs

c) 72 Hrs

d) 96 Hrs

152) A woman comes with postdated pregnancy at 42 weeks. The initial evaluation should be

a) USG

b) Induction of labor

c) Review previous menstrual history

d)

 

153) In pregnancy, the most common cause of transient Diabetes Insipidus is

a) Severe pre-eclampsia

b) Multiple pregnancy

c)

d)

154) A 26 year old nulliparous woman is on oral contraceptive pills. She is currently diagnosed as having pulmonary tuberculosis. Which anti-tuberculous drug decreases the effect of OCP's?

a) Rifampicin

b) INH

c) Pyrazinamide

d) Ethambutol

155) The drug that inhibits uterine contractility causing pulmonary edema is

a) Ritodrin

b )Nifedipine

c) indomethacin

d) Atabusin

156) Post-coital test detects all of the following except

a) Sperm count

b) Sperm abnormality

c) Fallopian tube block

d) Cervical factor abnormality

157) A 48 year old female suffering from severe menhorragia (DUB) underwent hysterectomy. She wishes to take hormone replacement therapy. Physical examination and breast are normal, but x ray shows osteoporosis. The treatment of choice is

a) Estrogen

b) Progesterone

c) Estrogen-Progesterone

d) None

158) A pregnant female, 38 years old, had a child with Downs syndrome. How do you assess the risk of Down's syndrome in the present pregnancy?

a) Chorionic villus biopsy

b) Maternal alpha feto protein levels

c) Maternal hCG

d) USG

159) A 28 year old female with a history of 8 weeks amenorrhea complains of vaginal bleeding and lower abdominal pain. On USG examination, there is gestational sac with absent fetal parts. The diagnosis is

a) corpus luteum cyst

b) ectopic pregnancy

c) Incarcerated abortion

d)Threatened abortion

160)Kamla, a 48 year old lady underwent hysterectomy. On the seventh day, she developed fever, burning micturiton and urinary dribbling . She can also pass urine voluntarily. The diagnosis is

a) uretero-vaginal fistula

b) vesico-vaginal fistula

c) Urge incontinence

d) Stress incontinence

161)A patient treated for infertility with clomiphene citrate presents with sudden onset of abdominal pain and distension with ascites. The probable cause is

a) Hyperstimulation syndrome

b) Uterine rupture

c) Ectopic pregnancy rupture

d) Multi fetal pregnancy

162) A woman at 8 months of pregnancy complains of abdominal pain and slight vaginal bleed. On examination, the uterine size is above the expected date with absent fetal heart sounds. The diagnosis is

a) Hydramnios

b)Uterine rupture

c) concealed hemorrhage

d) active labor

163) In a woman having a previous history of Caesarian section, all of the following are indications for trial labor except

a) Occipito posterior position

b) fetal distress

c) breech presentation

d) Mid pelvic contraction

164) All are indicators for the assessment of the nutritional program except

a) Weight and height of the preschool child

b) Prevalence of low Birth weight less than 2.5 kg in community

c) Nutritional assessment of the preschool child

d) Prevalence of pregnant mothers having Hb < 11.5 g% in the 3rd trimester

165) A concept directed against prevention of risk factors of CAD is

a) Primordial prevention

b) Secondary prevention

c) Health education

d) Primary prevention

166) In a community, the specificity of ELISA is 99% and sensitivity is 99%. The prevalance of the disease is 5/1000. Then, the positive predictive value of the test is

a) 33%

b) 67%

c) 75%

d) 99%

167) In a village of 1 lakh population, among 20000 exposed to smoking, 200 developed cancer , and among 40000 people unexposed, 40 developed cancer. The relative risk of smoking in the development of cancer is

a) 20

b) 10

c) 5

d)15

168) A person wants to visit a malaria endemic area of low level chloroquine resistant falciparum malaria. The best chemoprophylaxis is

a) Chloroquine

b) Proguanil + Chloroquine

c) Sulfadoxine + pyrimethamine

d) Mefloquine

169) A 35 year old male suffering from sudden onset of high grade fever. On malarial slide examination, all stages of the parasite are seen with schizonts of 20 microns size with 14-20 merozoites per cell and yellow brown pigment. The diagnosis is

a) Plasmodium falciparum

b) Plasmodium vivax

c) Plasmodium malariae

d) Plasmodium ovale

170) A child after consuming food in a party complaints of vomiting and diarrhea within 1-5 hours. The diagnosis is

a) Staphylococcus aureus

b) Streptococcus

c) Clostridium Perfringens

d) Clostridium Botulinum

171) Culex tritaenorrhyncus transmits

a) Dengue fever

b) Yellow fever

c) KFD

d) Japanese encephalitis

173) For the disposal of the hospital refuse, the bag made by cadmium is not used because incineration of the bag causes poisonous toxic fumes evolution. The color of the bag is

a) Black

b) Red

c) Blue

d) Yellow

174) Berkesonian bias refers to

a) Different rates of admission to the hospital

b) Interviewers bias

c)

d)

175) A 10 year old boy following a road traffic accident presents to the casualty with contaminated wound over the left leg. He has received his complete primary immunization before preschool age and received a booster of DT at school entry age. All of the following can be done except

a) Injection of TT

b) Injection of human antiserum

c) Broad spectrum antibiotics

d) Wound debridment and cleaning

176) A malarial survey is conducted in 50 villages having a population of one lakh. Out of 20,000 slides examined, 500 turned out to be malaria positive. The annual parasite index is

a) 20%

b) 5/1000 Population

c) 0.5%

d).4%

178) For a typhoid endemic country like India, the immunization of choice is

a) TAB Vaccine

b) typhoral 21A oral vaccine

c) monovalent vaccine

d)

179) Kallu, a 22 yr old male had an outing with his friends and developed fever of 38.5 degree C, diarrhea and vomiting following eating chicken salad. 24 hours back, two of his friends developed the same symptoms. The diagnosis is

a) Salmonella enteritis poisoning

b) Bacillus cereus

c) Staphylococcus aureus

d) Vibrio cholera

180) Simple randomization is done for

a) Every person has an equal and known chance of selection

b)

c)

d)

181) In a normal distribution curve, the true statement is

a) Mean = SD

b) Median =SD

c) Mean =2 Median

d)Mean = Mode

182)On prescription of oral pills to the user, the health worker will ask about the following except

a) Number of live children

b) calf tenderness

c) Headache

d) Swelling of the feet

183) A patient of paucibacillary tuberculoid leprosy completed 6 months of multidrug therapy. The response to therapy is good, but the lesion has not healed completely. According to the WHO criteria , which of the following should be done?

a) Stop treatment and watch

b) Continue the treatment for 6 more months

c) Continue Dapsone for 2 more months

d) Test for drug resistance

184) General fertility rate is a better measure of fertility than the crude birth rate because

the denominator includes

a) 15-45 years of age female.

b) Midyear population

c) Total woman population

d) Married woman population

185) A patient with sputum positive pulmonary tuberculosis is on ATT for the last 5 months but the patient is still positive for AFB in the sputum. This case refers to

a) New case

b) Failure case

c) Relapse case

d) Drug defaulter

186) Under the baby friendly hospital initiative program, all of the following can be done except

a)Breast feeding started 1-4 hours after birth

b) Mother and the child are kept together for 24 hours of the day

c) Feeding on demand

d) Exclusive breast feeding without any other food is preferred upto 4 months

187) In India. all are direct causes of maternal mortality except

a) Cardiac disease

b) Eclampsia

c) Hemorrhage

d) Abortion

188) In a group of 100 children, the weight of a child is 15 Kg. The standard error is 1.5 Kg.

Which one of the following is true

a) 95% of all children weigh between 12 and 18 Kg

b) 95% of all children weigh between 13.5 and 16.5 Kg

c) 99% of all children weigh between 12 and 18 Kg

d) 99% of all children weigh between 13.5 and 16.5 Kg

189) Malaria incidence in a village in the year 2000 is 430, 500, 410, 160, 270, 210, 300, 350, 4000, 430, 480, 540. Which of the following is the best indicator for assessment of malaria incidence in that village by the epidemiologist?

a) Arithmetic mean

b) Geometric mean

c) Median

d) Mode

190) In which of the following are granulomas not seen

a) Wegeners Granulomatosis

b) Giant cell arteritis

c) Microscopic polyangiiitis

d) Chrug-Strauss vasculitis


1)     A 65 year old female Kamala gives the history of severe unilateral headache on the right side and complains of blindness since 2 days .On examination there is a thick cord like structure on the lateral side of the head. The ESR is 80 mm/Hr in the first hour. The most likely diagnosis is

a) Temporal arteritis    b) Migraine   c) cluster headache  d) sinusitis

Ans (a)

2)     A 60 year old man complaining of chest pain since 6 hrs is diagnosed as acute MI . Angiography showed the involvement of anterior descending branch of left coronary artery The most probable site of involvement is

(a)   Anterolateral  b) Posterior c) Inferior d) Septal

Ans (a)

3) Pulmonary Kerley B lines are seen in all the following except

(a)   Pulmonary edema  b) Mitral valve disease c) Interstitial fibrosis d) Broncho nb alveolar Ca

Ans (d)

4) Infective endocarditis is least likely to occur in

(a)    ASD  b) Small VSD c) MVP d) TOF

Ans (a)

5)In a patient with chronic AF with a regular beat of 60/min, the most probable cause is

a)     sleep  b) digitalis toxicity  c) Sino nodal block  d) Hypothyroidism

Ans (b)

6)An HIV positive female has an indurated ulcer over the tongue. Laboratory findings show growth in cornmeal agar at 20 Degrees, microscopy showing hyphae and growth in human serum at 37 degrees show budding yeasts. The probable cause is

a)Candida albicans  b)Histoplasmosis  c)Blastomycosis  d) Coccidiodomycosis

Ans (a)

7) Kallu, a 30 year old man, presented with subcutaneous itchy nodules over the left iliac crest. On examination, they are firm, non tender and mobile .Skin snips contain microfilaria and adult worms of

a) Loa Loa  b) Onchocerca volvulus c) Brugia malayi d) Mansonella perstans

Ans (b)

8)In an ICU patient on invasive monitoring, all are causes of bacterial sepsis except

a)Orotracheal intubation  b)Intra arterial line c)Humidified air d)Central venous catheter

Ans (c)

9) In Polycythemia vera, all the following are seen except

a) thrombocytopenia b) increased GI bleed  c) thrombosis d) Transient visual loss

Ans (a)

10) A stem cell disorder affecting all the 3 cell lines - platelets, RBC’s and leucocytes is

a) Hemolytic anemia b) Paroxysmal cold haemoglobinuria c) PNH d) Blackfan Diamond syndrome

Ans (c)

11) In beta thalassemia, there is

a) increase in Beta chain , decrease in Alpha chain b) decrease in Beta chain, increase Alpha chain     c) decrease in Beta chain, decrease Alpha chain  d) Increase in Beta chain, increase Alpha chain

Ans (d)

12) Non caseating granulomas are seen in all the following except

a) Tuberculosis b) Byssinosis c) Hodgkin’s Lymphoma d) Metastatic carcinoma of lung

Ans (a)

13) All are paraneoplastic syndromes except

a) cerebellar degeneration b) Progressive Multifocal leukoencephalopathy c) Amyotropic lateral sclerosis d) Opsoclonus myoclonus

Ans (c)

14) In Burkitts lymphoma, translocation seen is

a) 12-14 translocation b) 8-14 translocation c)

Ans (b)

15) Ingestion of arsenic causes

a) Hepatic Ca  b) Hepatic adenoma c) Noncirrhotic portal fibrosis d) Hepatic cirrhosis

Ans (c)

16) In malignant hypertension, hyperplastic angiitis is seen in all except

a) Peripancreatic fat b) Kidney  c) Heart  d) Periadrenal fat

Ans (c)

17) In a chronic smoker, a highly malignant, aggressive and metastatic lung carcinoma is

a) Squamous cell Carcinoma b) Small cell Carcinoma c) Adenocarcinoma d) Large cell Carcinoma

Ans (b)

18) All are true about Neisseria gonorrhea except

a) Gram positive cocci  b) causes stricture urethra c) Involves seminal vesicles and spreads to epididymis  d) drug of choice is Ceftriaxone

Ans (a)

19) A 30 year old male, Kallu, with a history of sexual exposure comes with a painless indurated ulcer over the penis with everted margins. The diagnosis is

a) Syphilis  b) Chancroid ) LGV  d) Granuloma inguinale

Ans (a)

20) All are features of peripheral neuritis in a patient with Hansens Disease except

a) Predominant sensory involvement  b) Decreased tendon reflexes   c) Mutilations  d)

Ans (b)

21) A 32 year old male, Kallu, who recently visited a sea coast presented with an ulcer over the left leg. The probable cause is

(a)   Pasturella multocida b) Micrococcus halophilus c) Vibrio vulnificus d)

Ans (a)

22) An 8 year old boy, Kallu, is brought to the casualty with a history of consuming something while playing outside in a play ground. On examination, there was altered sensorium, hyperpyrexia, dilated pupils. Most probable cause is

a) Dhatura poisoning  b) Organophosphorus poisoning c) Parthenium poisoning d) Barbiturate poisoning

Ans (a)

23) All the following viruses cause stomach and colon cancer except

a) H pylori b) EB virus c) HBV  d) HIV

Ans (c)

24) Which of the following is pathognomonic of renal disease

a) Hyaline casts  b) coarse granular casts  c) cystine oxalate crystals d) epithelial cells

Ans (c)

25) Hypoglycemia is seen in

a) Acromegaly b) Cushings syndrome  c) Hypothyroidism  d) Hypopitutarism

Ans (d)

26) A 76 year old male comes with a history of frequent falls and difficulty in looking downwards and laterally. The diagnosis is

a) Alzheimer’s disease  b) Supranuclear palsy  c) Amyotropic lateral sclerosis d) Oculomotor nerve palsy

Ans (b)

27) The commonest side effect of cisplatin in a patient using it for esophageal carcinoma is

a) ATN b) Thrombocytopenia  c) Hepatic failure  d) Cardiomyopathy

Ans (a)

28) The most common late CNS complication of HIV is

a) Dementia  b) Ataxia  c) Seizures   d) Delirium

Ans (a)

29) In a patient with acute liver failure, the best prognostic indicator is

a) Serum albumin  b) Serum AFP  c) Serum bilirubin  d) Factor V estimation

Ans (d)

30) The commonest hepatotropic virus causing increased chronic carrier state is

a) HEV   b) HAV  c) HBV  d) HCV

Ans (d)

31) Regarding Addisonian pigmentation, all are true except

a) involves moles and scars  b) involves palmar creases  c) does not involve oral mucosa  d) involves sole pads

Ans (c)

32) In an HIV positive patient with tuberculosis, all are true except

a) decreased cavitation  b) increased sputum positivity  c) highly variable tuberculin test  d) decreased fibrosis

Ans (b)

33) In essential hypertension, changes seen in the heart are

a) cardiac cell hyperplasia  b) cardiac cell hypertrophy  c) increase in the mitochondrial number d)

Ans (b)

34) In a 32 year old male presenting with the following blood chemistry : Na+ 135, K+ 5.0, HCO3- 14.0, Cl- 116, PO4 5.0, SO4 5.0, Mg 2.0, Ca 8.0, the anion gap is

a) 10  b) 15 c) 13 d) 20

Ans (a)

35) Pyramidal tract involvement with absent ankle jerk is seen in

a) Frederick’s ataxia  b) Subacute combined degeneration of the spinal cord c) Lathyrism d) Tabes dorsalis

Ans (b)

36) A patient with BHP underwent TURP and developed altered sensorium. The cause is

a) Hyponatremia  b) Hypernatremia  c) Hypomagnesemia  d) Hypokalemia

Ans (a)

37) A patient of BHP underwent TURP under bupivacaine spinal anesthesia. One hour later, he developed nausea, vomiting and altered sensorium. The cause possibly is

a) Hypernatremia  b) Water intoxication  c) Bupivacaine overdose  d) Rupture Bladder

Ans (b)

38) Neuroblastoma differs from Wilm’s tumor radiologically by all except

a) same location  b) intraspinal extension  c) calcification  d) aorta and IVC are not eroded but pushed aside

Ans (d)

39) A 60 year old male, Kallu, on CT scan was found to have a solid tumor involving the inferior vena cava and renal vein. The tumor was within the Gerota’s fascia. All the following can be done except

a) chest X-ray to rule out pulmonary metastasis  b) pre-operative radiotherapy is not favorable  c) IVC invasion is inoperable d) pre-operative biopsy is not indicated

Ans (c)

40) A male patient, Kamal, aged 50 years presents with gradually progressive hard scrotal swelling on the right side. All the following can be done except

a) inguinal exploration b) chest Xray to rule out pulmonary metastasis c) CT abdomen   d) biopsy through the scrotal skin

Ans (d)

41) Orhcidectomy is done in all the following except

a) male breast cancer b) filarial epididymo-orchitis c) seminoma testes d) prostate cancer

Ans (b)

42) The treatment of choice in congenital hydrocele is

a) Eversion of the sac b) Excision of the sac  c) Herniotomy  d) Lord’s procedure

Ans (c)

43) In a patient with pheochromocytoma, all the following are seen except

a) diarrhea  b) orthostatic hypotension  c) episodic hypertension  d) weight gain

Ans (d)

44) Splenectomy is done in all the following except

a) splenic abscess  b) hereditary spherocytosis  c) sickle cell anemia  d) myelofibrosis

Ans (c)

45) A patient with recurrent attacks of cholelithiasis had, on ultrasound examination , a dilated CBD of 1 cm. The next best line in management is

a) ERCP b) PTC  c) intravenous cholangiogram  d) cholecystostomy

Ans (a)

46) A 40 year old male, Kallu, a chronic alcoholic, diagnosed as cirrhosis presents with a lump in the right lobe of the liver. Serum AFP levels are not elevated. The diagnosis is

a) fibrohyperplasia   b) hepatocellular carcinoma  c) hepatocellular adenoma   d)secondary from carcinoma colon

Ans (a)

47) A 60 year old male diagnosed to have carcinoma stomach had, on CT scan of the abdomen, a mass measuring 4x4 cm in the antrum with involvement of celiac nodes and right gastric nodes. The management of choice is

a) palliative b) subtotal gastrectomy  c) total gastrectomy  d) chemotherapy and radiotherapy

Ans (b)

48) A 70 year old male, Kamal, presents with lower GI bleed for the last 6 months. On sigmoidoscopic examination, there is a hard non-obstructing mass of 4 cms about 3 cms above the anal verge. The treatment of choice is

a) anterior resection b) abdominoperineal resection  c) defunctioning anastomosis               d) colostomy

Ans (b)

49) A 14 year old girl, Salu, a regular swimmer presents with sudden onset of pain abdomen, abdominal distension and fever of 390, but without obliteration of the liver dullness. The diagnosis is

a) ruptured typhoid ulcer  b) ruptured ectopic pregnancy  c) primary bacterial peritonitis    d) UTI with PID

Ans (c)

50) Left sided pulmonary hypertension is best treated by

a) splenectomy  b) central  spleno-renal shunt  c) portocaval shunt  d) distal lienorenal shunt

Ans (b)

51) A 28 year old male, Mallu, a chronic alcoholic, presents with sudden onset of epigastric pain that is radiating to the back. All the following can be seen except

a) hypocalcemia b) increased serum amylase  c) low serum lipase  d) increased LDH

Ans (c)

52) A 28 year old male patient is brought to the casualty with severe hypotension following a car accident. On examination no external bleed or injuries are seen. The cause of hypotension is possibly

a) intrathoracic and abdominal bleed  b) fracture rib c) intracranial bleed  d) neurogenic shock

Ans (a)

53) On prenatal ultrasound examination, the diagnostic feature of congenital diaphragmatic hernia is

a) absence of gas bubble under the diaphragm  b) mediastinal shift with normal heart axisc) peristalsis in the thoracic cavity  d) a cyst behind the left atrium

Ans (c)

54) A 40 year old male, Kallu, a chronic smoker, presents with claudication and a medial leg ulcer. For the past one month, he gives a history of rest pain. All the following can be used to relieve the rest pain except

a) Omentoplexy   b) conservative amputation  c) lumbar sympathectomy  d) femoropopliteal bypass

Ans (c)

55) A male patient aged 30 years following a road traffic accident presents with fracture of 4th to 10th ribs and respiratory distress. He is diagnosed to have flail chest and PaO2 is < 60%. The treatment of choice is

a) fixation of ribs b) strapping of the chest  c) IPPV with oral intubation  d) tracheostomy

Ans (c)

56) A post-operative patient presents with duodenal leak and peritonitis with massive contamination. The appropriate management of choice is

a) duodenostomy + feeding jejunostomy + peritoneal lavage  b) total parenteral nutritionc) duodenojejunostomy d) four quadrant peritoneal lavage

Ans (a)

57) A 52 year old male executive is seen in the casualty with hypotension, vomiting bright red blood at home and in the hospital. There is no previous suggestive history. The estimated blood loss is around 2 liters. The diagnosis is

a) esophageal varices  b) gastritis   c) duodenal ulcer  d) Mallory-Weiss tear

Ans (c)

58) All are true about the right kidney except

a) it is related to the duodenum  b) it is lower than the left kidney  c) the right renal vein is shorter than the left  d) right kidney is preferred over the left for transplantation

Ans (d)

59) Grade IV esophageal varices, on barium swallow appears as

a)     a thick band  b) mucosal folds above the carina  c) mucosal folds below the carina 

d)

Ans (?)

60) A female patient presents with pigmentation of the lips and oral mucosa and colonic polyps. Her sister also has the same history. The diagnosis is

a) Peutz-Jeghers’ syndrome  b) Carcinoid  c) melanoma  d) villous adenoma

Ans (a)

61) The commonest site of carcinoma prostate is

a) Transitional zone  b) Central zone  c) Peripheral zone  d) Anterior zone

Ans (c)

62) Dissociative sensory loss is seen with

a) Tabes dorsalis  b) Syringomyelia  c)

Ans (b)

63) In a patient with raised IgA levels, the commonest finding is

a) proteinuria b) GI bleed  c) hematuria  d) hypertension

Ans (c)

64) In a patient with choledochal cyst, bile diversion into the small intestine is contraindicated because of risk of

a) malignancy  b) recurrent cholangitis  c) pancreatitis  d) increased gall stones

Ans (a)

65) Thyrotoxicosis differs from malignant hyperthermia by

a) muscle rigidity  b) CPK  c) hypothermia  d)

Ans (b)

66) In a female with sudden onset of severe headache, on CT, a diagnosis of subarachnoid hemorrhage was made. The most common site of subarachnoid hemorrhage is

a) subdural venous sinuses b) middle meningeal artery  c) Berry aneurysm rupture  d) basilar artery

Ans (c)

67) Chamavati, a 30 year old female from rural Assam with a history of chronic tobacco chewing since 15 years of age, presents with difficulty in opening the mouth. On oral examination, no ulcer is seen. The diagnosis is

a) non-ulcerative carcinoma of buccal mucosa  b) submucous oral fibrosis  c)Temporomandibular joint arthritis

Ans (b)

68) A 40 year old female patient, Chamavati, presents with a lump that is gradually increasing in the parotid region. On oral examination, the tonsil was pushed medially. On biopsy, pleomorphic adenoma was diagnosed. The appropriate treatment is

a) Superficial parotidectomy b) Lumpectomy  c) Enucleation  d) Conservative total dissection

Ans (a)

69) A female patient, 45 years of age, with a family history of breast carcinoma showed diffuse microcalcification on mammography. On biopsy, intraductal carcinoma in situ was found. The appropriate management is

a) simple mastectomy  b) quadrantectomy  c) radical mastectomy with axillary sampling  d) chemotherapy with CMF regime

Ans (c)

70) Tram track appearance on CT scan of the head is seen in

a) Sturge Weber syndrome  b) von Hippel Lindau disease  c) Tuberous sclerosis   d) neurofibroma

Ans (a)

71) A patient presents with minimal pleural effusion on the right side. The best method to detect this would be

a) right side chest Xray b) Left sided chest Xray  c) Left lateral decubitus chest Xray  d) Right lateral decubitus chest Xray

Ans (d)

72) A 40 year old farmer with a history of recurrent attacks of porphyria complains of itching when exposed to the sun and maculopapular rash on sun-exposed areas. His symptoms are exaggerated in the summer. The diagnosis is

a) Seborrheic dermatitis b) Contact dermatitis  c) Psoriasis  d) Porphyria cutanea tarda

Ans (d)

73) An 8 year old boy presents with a well defined annular lesion over the buttock with central scarring that is gradually progressing over the last 8 months. The diagnosis is

a) annular psoriasis  b) lupus vulgaris  c) tinea corporis  d)

Ans (b)

74) In adult polycystic kidney, all are true except

a)   Cysts are seen in the liver, spleen and the pancreas b) Hematuria occurs 

c) Hypertension is rare                     d) Autosomal dominant transmission is seen

Ans (c)

75) In thymoma, all are seen except

a) red cell aplasia b) hyperalbuminemia  c) hypogammaglobulinemia  d) myasthenia gravis

Ans (b)

76) An adult presents with oval scaly hypopigmented macules over the chest and the back. The diagnosis is

a) Leprosy  b) Lupus vulgaris  c) Pityriasis versicolor  d) Lichen planus

Ans (c)

77) The characteristic nail finding in lichen planus is

a) pitting b) pterygium  c) Beau’s lines  d) Hyperpigmentation of the nails

Ans (b)

78) In an 8 day old child with no history of consanguinity in the parents, the mother reports blisters and peeling off of the skin at the site of handling and pressure. There was a similar history in the previous child which proved to be fatal. The diagnosis is

a)Bullous pemphigoid  b) Congenital syphilis  c) Congenital epidermolysis bullosa   d)etterer-Siwe disease

Ans (b/d)

79) The most common complication of hypermature sclerotic cataract is

a) dislocation of the lens  b) phakomorphic glaucoma  c) uveitis  d)

Ans (a)

80) A 14 year old boy complains of pain during reading . On examination, his both eyes are normal and vision with non Snellen’s reading is 6/5. He still complains of pain on occluding one eye. The diagnosis is

a) myopia  b) pseudomyopia  c) hyperopia  d) emmetropia

Ans (b)

81) A 16 year old boy complains of pain in the right eye. After refractometry , he was prescribed a + 3.5 D sphere lens. The cover test is normal. There is no heterophoria. The diagnosis is

a) organic amblyopia  b) anisometric amblyopia  c) emmetropic amblyopia  d) toxic amblyopia

Ans (?)

82) A patient complains of pain in both eyes with congestion, blurring of vision, photophobia and mucopurulent discharge since one day. Many cases have been reported from the same community. The causative agent is probably

a) adenovirus  b) enterovirus 70  c) herpes simplex  d)

Ans (b)

83) A male patient with a history of hypermature cataract presents with a 2 day history of ciliary congestion, photophobia, blurring of vision and on examination has a deep anterior chamber in the right eye. The left eye is normal. The diagnosis is

a) phakomorphic glaucoma  b) phakolytic glaucoma  c) phakotoxic glaucoma     d)phakoanaphylactic uveitis

Ans (d)

84) A 60 year old male patient operated for cataract 6 months back now complains of floaters and sudden loss of vision. The diagnosis is

a) vitreous hemorrhage b) retinal detachment  c) central retinal artery occlusion   d) cystoid macular edema

Ans (a)

85) A 12 year old boy presents with recurrent attacks of conjunctivitis for the last 2 years with intense itching and ropy discharge. The diagnosis is

a) vernal conjunctivitis  b) phlyctenular conjunctivitis  c) trachoma  d) viral conjunctivitis

Ans (a)

86) A 25 year old lady presents with severe sudden onset of pain, corneal congestion, photophobia and deep anterior chamber in the right eye. The left eye is normal. Xray pelvis shows sacroiliitis. The diagnosis is

a) anterior uveitis  b) posterior uveitis  c) intermediate uveitis  d) scleritis

Ans (a)

87) A 30 day old neonate was presented with a history of photophobia and excessive lacrimation. On examination, both the lacrimal duct systems are normal, but there was a large cornea and corneal haziness. The diagnosis is

a) megalocornea  b) keratoconus  c) congenital glaucoma  d) Hunter’s syndrome

Ans (c)

88) In high spinal anesthesia, seen are

a) hypotension and bradycardia    b) hypotension and Tachycardia  c) hypertension and bradycardia            d) hypertension and tachycardia

Ans (a)

89) A patient selected for surgery who was induced with thiopentone i.v through one of the antecubital veins complains of severe pain of the whole hand. The next line of management is

a) give i.v propofol through the same needle  b) give i.v ketamine through the same needle  c) give i.v lignocaine through the same needle  d) leave it alone

Ans (c)

90) A patient who was on aspirin for a long period was selected for an elective surgery. What should be done?

a) stop aspirin for 7 days b) infusion of fresh frozen plasma  c) infusion of platelet concentrate  d) go ahead with the surgery maintaining adequate hemostasis

Ans (a)

91) A patient in the ICU was on invasive monitoring with intraarterial cannulation through the right radial artery for the last 3 days. Later, he developed swelling and discoloration of the right hand. The next line of management is

a) stellate ganglion block  b) brachial block  c) radial nerve block on the same side  d) application of lignocaine jelly over the site

Ans (a)

92) A 3 year old child with severe sensorineural deafness was prescribed hearing aids, but shows no improvement. The next line of management is

a) cochlear implant  b) fenestration surgery  c) stapes mobilization  d)

Ans (a)

93) A 4 year old girl diagnosed as having multiple juvenile papillamotosis of the larynx presents to the casualty with mild respiratory distress. The next line of management is

a) tracheostomy  b) microlaryngoscopy  c) broad spectrum antibiotics  d) systemic steroids

Ans (b)

94) A 28 year old female presents to the casualty with gradually increasing respiratory distress since 4 days. She gives history of hospitalization and mechanical ventilation with orotracheal intubation for 2 weeks. Now, she was diagnosed as having severe tracheal stenosis. The next line of management is

a) tracheal resection and end to end anastomosis  b) tracheal dilatation  c) systemic steroids                   d)laser excision and stent insertion

Ans (d)

95) A 30 year old male, following excision of the 3rd molar for dental caries presents with trismus, fever of 39.50 and swelling pushing the tonsil medially and spreading laterally posterior to the middle sternocleidomastoid. The diagnosis is

a) parapharyngeal abscess  b) retropharyngeal abscess  c) Ludwig’s angina  d) submental abscess

Ans (a)

96) A 20 year old male, following a road traffic accident was brought to the casualty. His right leg is shortened, internally rotated and adducted. The diagnosis is

a) fracture neck of femur b) anterior dislocation of the hip  c) posterior dislocation of the hip                       d) trochanteric fracture of the femur

Ans (c)

97) An 8 year old boy presents with a gradually progressing swelling and pain since 6 months over the upper tibia. On Xray, there is a lytic lesion with sclerotic margins in the upper tibial metaphysis. The diagnosis is

a) Osteogenic sarcoma  b) Osteoclastoma  c) Brodie’s abscess  d) Ewing’s sarcoma

Ans (c)

98) An 8 year old boy with a history of fall from 10 feet height complains of pain in the right ankle. Xrays taken at that time are normal without any fracture line. But after 2 years, he developed a calcaneovalgus deformity. The diagnosis is

a) undiagnosed malunited fracture  b) avascular necrosis talus  c) tibial epiphyseal  injury

Ans (c)

99) A 6 year old boy has a history of recurrent dislocation of the right shoulder . On examination, the orthopedician puts the patient in the supine position and abducts his arm to 90 degrees with the bed as the fulcrum and then externally rotates it but the boy does not allow the test to be performed. The test done by the orthopedician is

a) apprehension test  b) sulcus test  c)

Ans (a)

100) Regarding bone remodelling, all are true except

a) osteoclastic activity at the compression site b) osteoclastic activity at the tension site   c) osteoclastic activity and osteoblastic activity are both needed for bone remodelling in cortical and cancellous bones  d) osteoblasts transforms into osteocytes

Ans (?)

101) A child is spinned around by his father by holding both hands. While doing this the child started crying and does not allow his father to touch his elbow.The diagnosis is

a) pulled elbow  b) radial head dislocation  c) Annular ligament tear  d) Fracture olecranon process

Ans (a)

102)The structures piercing the clavipectoral fascia are all except:

a) Cephalic vein  b) Thoraco acromial artery  c) Lateral pectoral nerve  d) Lateral pectoral artery

Ans (d)

103) Crossed oculomotor palsy is seen in occlusion of

a) Anterior cerebral artery  b) Middle cerebral artery  c) Posterior cerebral artery               d) Superior cerebral artery

Ans (c)

104) Erection of penis is mediated by all the following except

a) Nervi erigentes  b) Pudendal nerve  c) Sacral plexus  d) Hypogastric plexus

Ans (d)

105) Active transport of chemicals across the cell membrane is mediated by:

a) Channel protein  b) Carrier protein  c) G protein   d) Na+- K+ ATPase.

Ans (b)

106) In a patient with respiratory rate of 14/min, tidal volume of 500 ml with a vital capacity 7000ml, the alveolar ventilation/minute is

a) 2000 ml  b) 4900ml  c) 7000ml  d) 7700ml.

Ans (b)

107) During sigmoidoscopy, if the rectum is inflated with gas, increased peristalsis is seen in

a) Whole intestine  b) Distal colon  c) Proximal colon  d) Whole colon

Ans (?)

108) In an unacclimatised person suddenly exposed to cold, the physiological effect seen is

a) Hypertension  b) Tachycardia  c) Shift of blood from shell to core  d) Non shivering thermogenesis

Ans (?)

109) Spuriously high BP is noted in all the following except

a) Obesity  b) Thick calcified vessels  c) Small cuff  d) Auscultatory gap

Ans (a)

110) Cerebellar herniation presents with all the following except

a) Loss of consciousness  b) Autonomic disturbances  c) Neck stiffness  d) Pupil dilatation

Ans (c)

111) Positive feedback is seen in all the following except

a) LH Surge b) Stimulation of the gastric secretion by histamine and gastrin  c) thrombolytic activity in the coagulation cascade  d) Entry of Ca into the sarcoplasmic reticulum

Ans (d)

112)Vitamin K is needed for the post translational modification of

a) Carboxylation  b) Methylation  c) Hydroxylation  d) Transketolation

Ans (a)

113) Amber codon refers to

a) Initiating codon  b) Mutant codon  c) Stop codon  d) Codon coding for multiple amino acids

Ans (?)

114) At physiological pH, the most stable amino acid is

a) Histidine b) Lysine  c) Arginine  d) Leucine

Ans (a)

115) In cystinuria, amino acids excreted are all the following except:

a) Ornithine  b) Arginine   c Lysine  d) Histidine

Ans (d)

116) Dietary triglycerides are transported by

a) Chylomicrons  b) LDL  c) VLDL  d) HDL

Ans (a)

117) In which of the following reaction, thiamine is not used

a) Alpha ketoglutarate to succinyl CoA  b) Glucose to pentose  c) Oxidative decarboxylation of Alpha keto amino acids  d) Lactate to pyruvate

Ans (d)

118) In chromatography, mass movement of the substances is seen in

a) Electrophoresis  b) Diffusion  c) Osmosis  d) Paper chromatography

Ans (a)

119) The type of chromatography in which proteins are bound to another substance is

a) Hydrophobic chromatography  b) Absorption ( Affinity ) chromatography

Ans (b)

120) The end-product of citric acid cycle used in detoxification of ammonia in brain is

a) Oxaloacetate  b) Alpha keto glutarate  c) Succinate   d) Citrate

Ans (b)

121) Right parietal lobe lesions cause all the following except

a) ideomotor apraxia  b) Self neglect 

Ans (?)

122) Investigation of choice for blood grouping in old blood stain on the cloth is

a) Precipitin test  b) Benzidine test  c) Acid dilution test  d) Hemin crystals

Ans (d)

123) Interleukin secreted by the macrophages stimulating lymphocytes is

a) IL 1  b) INF Alpha  c) TNF Alpha  d) IL 6

Ans (a)

124) Following injection of lymphokines, the same class of immunoglobulins are produced. This is referred to as

a) Clonal selection  b) Class switching  c)  d)

Ans (a)

125) A patient receiving allopurinol requires dose reduction of

a) 6 mercapto purine  b) cyclophosphamide  c) Azathioprine  d) Cimetidine

Ans (a)

126) An elderly hypertensive has diabetes mellitus and bilateral renal artery stenosis. The best management is

a) Enalapril   b) Verapamil  c) Beta blockers  d) Thiazides

Ans (a)

127) A female suffering from psychosis, taking phenothiazines now complains of sudden onset of high grade fever, muscle rigidity and altered sensorium. The diagnosis is

a) Malignant hyperthermia  b) Neuroleptic malignant syndrome  c) Tardive dyskinesia        d)akathesia

Ans (b)

128) A patient on cisapride for Barrets ulcer suffers from pneumonia. The physician prescribes erythromycin. Which of the following is the correct statement

a) Increase the dose of cisapride needed  b) Increase the dose of erythromycin needed  c) Increased risk of Ventricular arrythmias present  d) Decreased bioavailability of cisapride

Ans (c)

129) A 60 year old male comes to the casualty with acute retention of urine since 12 hours. On examination there was distended bladder . His son gives a history of taking some drugs by the patient since 2 days as he is suffering from depression . The most likely drug is

a) Amitryptiline  b) Chlorpromazine  c) Haloperidol  d)

Ans (a)

130) Kallu, a 22 year old male suffers from decreased sleep, increased sexual activity , excitement and spending excessive money excessively for the past 8 days. The diagnosis is

a) Acute mania   b) Acute psychosis  c) Schizophrenia  d) Psychosexual disorder

Ans (a)

131) Rathi, a 26 year old female diagnosed to be suffering from depression now for the past 2 days had suicidal tendencies, thoughts and ideas. The best treatment is

a) Amitryptiline  b) Selegiline    c) Haloperidol + Chlorpromazine  d) ECT

Ans (d)

132) Kallu, a 22 year old single unmarried man is suffering from sudden onset of 3rd person hallucinations for the past 2 weeks. He is suspicious of his family members and had decreased sleep and appetite. The diagnosis is

a) Schizophrenia  b) Acute psychosis  c) Acute mania  d) Delirium

Ans (b)

133) A 60 year old male suffering from auditory hallucinations is says that people staying upstairs are talking about him and conspiring against him. He dropped a police complaint against them but the allegations were proved to be wrong. The diagnosis is

a) Schizophrenia  b) Depression   c) Dementia  d) Delusional disorder

Ans (a)

134) In India the commonest cause of juvenile onset of DM is

a) MODY  b) Fibrocalcific pancreatopathy  c) Gall stones   d) IDDM

Ans (b)

135) A neonate has central cyanosis and short systolic murmur on the 2nd day of birth. The diagnosis is

a) TGV  b) TOF  c) VSD   d) ASD

Ans (a)

136) An 8 year old female child following URTI developed maculopapular rash on the face spreading onto the trunk which cleared on the 3rd day without desquamation and tender post auricular and suboccipital lymphadenopathy. The diagnosis is

a) Measles   b) Rubella    c) Erythema infectiosum  d) Kawasaki disease(Mucocutaneous LN Syndrome)

Ans (b)

137) A 4 month old HIV positive child following URTI ,developed sudden onset of breathlessness. The chest Xray shows hyperinflation. The O2 saturation was greater than 90%. The treatment of choice is

a) Nebulized acyclovir  b) i.v Ganciclovir  c) Ribavirin  d) Cotrimoxazole

Ans (c)

138) All of the following are features of systemic juvenile Rheumatoid arthritis except

a) Rash  b) fever  c) Hepato Splenomegaly  d) Uveitis

Ans (d)

139)A child climbs with alternate steps , builds a tower of 8-9 cubes , tells “I” but not his name and cannot say his age and sex. The probable age is

a) 24 Months  b) 36 Months  c) 30 Months  d) 48 Months

Ans (c)

140) A child suffering from acute diarrhea is brought to the casualty and is diagnosed as having severe dehydration with pH of 7.23 , Serum Na -125 , Serum K- 3 , Hco3 16. The best IV fluid of choice is

a) 3% saline  b) Normal saline  c) N/3 saline + 10 % dextrose  d) N/3 saline + 5% dextrose

Ans (?)

141) A child with recent onset of URTI after 2 days presents with acute onset of breathlessness, cough and fever. All of the following can be given except

a) Antibiotics  b) Antipyretics  c) O2 inhalation  d) Morphine

Ans (d)

142)A 6 month old child having severe dehydration comes to the casualty with weak pulse and unrecordable BP. Repeated attempts in gaining IV access has failed. The next best step is

a) Venesection  b) Jugular vein catheterization  c) Intraosseous IV Fluids  d) Try again

Ans (c)

143) A 6 year old child with acute onset of fever of 104 F developed febrile seizures and was treated. To avoid future recurrence of seizure attacks what should be given

a) IV diazepam infusion over 12 Hrs b) Paracetamol 400 mg + Phenobarbitone daily     c)pararacetamol 400 mg 6th Hrly  d) Oral diazepam 6th Hrly

Ans (c)

144 ) An 8 year old child suffering from recurrent attacks of polyuria since childhood presents to the paediatrics OPD. On examination, the child is short statured, vitals and BP are normal. Serum Cr 6 Mg %, HCO3 16 mEq , Na 134, K 4.2 On USG bilateral small kidneys The most likely diagnosis is

a) Polycystic Kidney disease  b) Medullary cystic Kidney disease  c) Nephronophthisis  d) Reflux nephropathy

Ans (c)

145) A 40 year old male presents with recurrent bouts of vomiting since 9 months because of  pyloric obstruction. The compensatory biochemical change is

a) Respiratory acidosis b) Respiratory alkalosis  c) Metabolic acidosis  d) Paradoxical aciduria with hypo natremia and hypo chloremia

Ans (d)

146)A diabetic female at 40 weeks of pregnancy delivered a baby by elective CS. Soon after birth the baby developed respiratory distress. The diagnosis is

a) Hyaline membrane disease  b) Transient tachypnea of the newborn  c) Congenital diaphragmatic hernia   d) Tracheo esophageal fistula

Ans (a)

147)All are prognostic indicators of PIH except

a) Serum Uric acid  b) Low platelets   c) Serum Na   d) Elevated liver enzymes

Ans (c)

148) In a woman on subdermal progesterone implant , the menstrual abnormality seen is

a) Amenorrhea  b) Menorrhagia  c) Metrorrhagia  d) Polymenorrhoea

Ans (c)

149) Kalavathi ,a 29 year old nulliparous woman complains of severe menorrhagia and lower abdominal pain since 3 months. On examination, there is a 14 weeks size uterus with fundal fibroid . The treatment of choice is

a) Wait and watch   b) Myomectomy  c) GnRH analogues  d) Hysterectomy

Ans (c)

150) A pregnant lady in the first trimester presented with random blood glucose of 177 mg /dl   The treatment is

a) Insulin   b) glipizide  c) Phenformin  d) Sulfonylurea

Ans (a)

151) In a non-diabetic high risk pregnancy, the ideal time for NST monitoring is

a) 24 Hrs   b) 48 Hrs   c) 72 Hrs  d) 96 Hrs

Ans (?)

152) A woman comes with postdated pregnancy at 42 weeks. The initial evaluation should be

a) USG  b) Induction of labor  c) Review previous menstrual history  d)

Ans (c)

153) In pregnancy, the most common cause of transient Diabetes Insipidus is

a) Severe pre-eclampsia  b) Multiple pregnancy  c)   d)

Ans (a)


154) A 26 year old nulliparous woman is on oral contraceptive pills. She is currently diagnosed as having pulmonary tuberculosis. Which anti-tuberculous drug decreases the effect of OCP’s?

a) Rifampicin  b) INH  c) Pyrazinamide  d) Ethambutol

Ans (a)

155) The drug that inhibits uterine contractility causing pulmonary edema is

a) Ritodrin  b )Nifedipine  c) indomethacin  d) Atabusin

Ans (a)

156) Post-coital test detects all of the following except

a) Sperm count  b) Sperm abnormality   c) Fallopian tube block  d) Cervical factor abnormality

Ans (c)

157) A 48 year old female suffering from severe menhorragia (DUB) underwent hysterectomy. She wishes to take hormone replacement therapy. Physical examination and breast are normal, but x ray shows osteoporosis. The treatment of choice is

a) Estrogen  b) Progesterone  c) Estrogen-Progesterone  d) None

Ans (a/c)

158) A pregnant female, 38 years old, had a child with Downs syndrome. How do you assess the risk of Down’s asyndrome in the present pregnancy?

a) Chorionic villus biopsy  b) Maternal alpha feto protein levels  c) Maternal hCG  d) USG

Ans (a)

159) A 28 year old female with a history of 8 weeks amenorrhea complains of vaginal bleeding and lower abdominal pain. On USG examination, there is gestational sac with absent fetal parts. The diagnosis is

(a)   corpus luteum cyst    b) ectopic pregnancy   c) Incarcerated abortion   d)Threatened abortion

Ans (c)

160)Kamla, a 48 year old lady underwent hysterectomy. On the seventh day, she developed fever, burning micturiton and urinary dribbling . She can also pass urine voluntarily. The diagnosis is

a) uretero-vaginal fistula   b) vesico-vaginal fistula  c) Urge incontinence  d) Stress incontinence

Ans (a)

161)A patient treated for infertility with clomiphene citrate presents with sudden onset of abdominal pain and distension with ascites. The probable cause is

a) Hyperstimulation syndrome  b) Uterine rupture   c) Ectopic pregnancy rupture   d) Multi fetal pregnancy

Ans (a)

162) A woman at 8 months of pregnancy complains of abdominal pain and slight vaginal bleed. On examination, the uterine size is above the expected date with absent fetal heart sounds. The diagnosis is

a) Hydramnios  b)Uterine rupture   c) concealed hemorrhage  d) active labor

Ans (c)

163) In a woman having a previous history of Caesarian section, all of the following are indications for trial labor except

a) Occipito posterior position  b) fetal distress   c) breech presentation   d) Mid pelvic contraction

Ans (b)

164) All are indicators for the assessment of the nutritional program except

a) Weight and height of the preschool child  b) Prevalence of low Birth weight less than 2.5 kg in community  c) Nutritional assessment of the preschool child  d) Prevalence of pregnant mothers having Hb < 11.5 g% in the 3rd trimester

Ans (d)

165) A concept directed against prevention of risk factors of CAD is

a) Primordial prevention   b) Secondary prevention  c) Health education  d) Primary prevention

Ans (a)

166) In a community, the specificity of ELISA is 99% and sensitivity is 99%. The prevalance of the disease is 5/1000. Then, the positive predictive value of the test is

a) 33%  b) 67%    c) 75%  d) 99%

Ans (a)

167) In a village of 1 lakh population, among 20000 exposed to smoking, 200 developed cancer , and among 40000 people unexposed, 40 developed cancer. The relative risk of smoking in the development of cancer is

a) 20      b) 10        c) 5         d)15

Ans (b)

168) A person wants to visit a malaria endemic area of low level chloroquine resistant falciparum malaria. The best chemoprophylaxis is

a) Chloroquine  b) Proguanil + Chloroquine  c) Sulfadoxine + pyrimethamine  d) Mefloquine

Ans (d)

169) A 35 year old male suffering from sudden onset of high grade fever. On malarial slide examination, all stages of the parasite are seen with schizonts of 20 microns size with 14-20 merozoites per cell and yellow brown pigment. The diagnosis is

a) Plasmodium falciparum  b) Plasmodium vivax  c) Plasmodium malariae  d) Plasmodium ovale

Ans (b)

170) A child after consuming food in a party complaints of vomiting and diarrhea within 1-5 hours. The diagnosis is

a) Staphylococcus aureus  b) Streptococcus  c) Clostridium Perfringens  d) Clostridium Botulinum

Ans (a)

171) Culex tritaenorrhyncus transmits

a) Dengue fever  b) Yellow fever  c) KFD  d) Japanese encephalitis

Ans (b)

172) For the disposal of the hospital refuse, the bag made by cadmium is not used because incineration of the bag causes poisonous toxic fumes evolution. The color of the bag is

a) Black  b) Red   c) Blue  d) Yellow

Ans (?)

173) Berkesonian bias refers to

a) Different rates of admission to the hospital   b) Interviewers bias 

Ans (?)

174) A 10 year old boy following a road traffic accident presents to the casualty with contaminated wound over the left leg. He has received his complete primary immunization before preschool age and received a booster of DT at school entry age. All of the following can be done except

a) Injection of TT  b) Injection of human antiserum   c) Broad spectrum antibiotics  d) Wound debridment and cleaning

Ans (a)

175) A malarial survey is conducted in 50 villages having a population of one lakh. Out of 20,000 slides examined, 500 turned out to be malaria positive. The annual parasite index is

a) 20% b) 5/1000 Population  c) 0.5%  d).4%

Ans (b)

176) For a typhoid endemic country like India, the immunization of choice is

a) TAB Vaccine  b) typhoral 21A oral vaccine  c) monovalent vaccine   d)

Ans (b)

177) Kallu, a 22 yr old male had an outing with his friends and developed fever of 38.5 degree C, diarrhea and vomiting following eating chicken salad. 24 hours back, two of his friends developed the same symptoms. The diagnosis is

a) Salmonella enteritis poisoning  b) Bacillus cereus   c) Staphylococcus aureus   d) Vibrio cholera

Ans (?)

178) Simple randomization is done for

a) Every person has an equal and known chance of selection 

Ans (c)

179) In a normal distribution curve, the true statement is

a) Mean = SD  b) Median =SD  c) Mean =2 Median  d)Mean = Mode

Ans (a)

180)On prescription of oral pills to the user, the health worker will ask about the following except

a) Number of live children  b) calf tenderness  c) Headache  d) Swelling of the feet

Ans (a)

181) A patient of paucibacillary tuberculoid leprosy completed 6 months of multidrug therapy. The response to therapy is good, but the lesion has not healed completely. According to the WHO criteria , which of the following should be done?

a) Stop treatment and watch b) Continue the treatment for 6 more months  c) Continue Dapsone for 2 more months   d) Test for drug resistance

Ans (a)

182) General fertility rate is a better measure of fertility than the crude birth rate because the denominator includes

a) 15-45 years of age female. b) Midyear population  c) Total woman population  d) Married woman population

Ans (a)

183) A patient with sputum positive pulmonary tuberculosis is on ATT for the last 5 months but the patient is still positive for AFB in the sputum. This case refers to

a) New case  b) Failure case   c) Relapse case  d) Drug defaulter

Ans (a)

184) Under the baby friendly hospital initiative program, all of the following can be done except

a)Breast feeding started 1-4 hours after birth   b) Mother and the child are kept together for 24 hours of the day  c) Feeding on demand   d) Exclusive breast feeding without any other food is preferred upto 4 months

Ans (a)

185) In India. all are direct causes of maternal mortality except

a) Cardiac disease  b) Eclampsia  c) Hemorrhage  d) Abortion

Ans (5)

186) In a group of 100 children, the weight of a child is 15 Kg. The standard error is 1.5 Kg.Which one of the following is true

a) 95% of all children weigh between 12 and 18 Kg   b) 95% of all children weigh between 13.5 and 16.5   Kg   c) 99% of all children weigh between 12 and 18 Kg   d) 99% of all children weigh between 13.5 and 16.5 Kg.

Ans (a)

187) Malaria incidence in a village in the year 2000 is 430, 500, 410, 160, 270, 210, 300, 350, 4000, 430, 480, 540. Which of the following is the best indicator for assessment of malaria incidence in that village by the epidemiologist?

a) Arithmetic mean  b) Geometric mean  c) Median  d) Mode

Ans (a)

188) In which of the following are granulomas not seen

a) Wegeners Granulomatosis  b) Giant cell arteritis  c) Microscopic polyangiiitis  d) Chrug-Strauss vasculitis

 Ans (a)

 

1. A 52 yo AA hypertensive, diabetic, hypercholesterolemic man awakens and can not express himself. His entire right side is numb and weak. Neurologic exam shows a right hemiparesis (face, arm and leg are equal). Hemianesthesia is present and there is a Broca aphasia. The patient reports mild left temporal headache.




2. A 46 yo caucasian woman develops right sided visual blurring, "disorientation," and left sided throbbing temporal headache without vomiting. She then develops right hand and facial numbness. Her past history is significant for hypercholesterolemia, angina treated with percutaneous coronary angioplasty 10 years ago, and she was recently started on Premarin (conjugated estrogens) for hot flashes. The family history is negative for stroke or migraine.

Neurologic exam reveals a right homonymous hemianopsia, right hemianesthesia (face and arm are equal, the leg is not as affected). The blood pressure is 140/90, HR=80. The cardiac, skin, lung exams are normal. Homan's sign is absent, there are no bruits.

Imaging studies

The total cholesterol is 231 with elevated LDL and decreased HDL.

During her course the headache resolves with Tylenol and 4 days later the neurologic exam is normal.


18. A 67 year old hypertensive, diabetic, hypercholesterolemic, obese, sedentary man with prior history of coronary artery disease, peripheral vascular disease, and transient ischemic attack develops left facial numbness, vertigo and gait disequilibrium. This develops four days after an episode of emotional stress during which he is accused of causing a labor dispute involving his labor union.

Medications include Metformin (glucophage), Pravachol (pravastatin), metoprolol, verapamil, a thiazide diuretic, Imdur (isosorbide mononitrate), and Catapres (clonidine).

The past history is significant for transient ischemic attack, intermittent claudication and unstable angina.

The physical exam reveals BP 200/110, pulse 80. The neurologic exam reveals a broad based gait and a left Horner's syndrome. There is left facial and right body anesthesia, left dysmetria, decreased gag and palatal reflex, and dysarthria.


24. A 48 year old diabetic hypercholesterolemic woman has multiple transient right eye visual loss episodes (duration 15 minutes). She described these "as if a curtain is being pulled down." There are no other accompanying symptoms. The past history is significant for coronary artery disease with prior myocardial infarction, peripheral vascular disease with intermittent claudication, migraine with aura as an adolescent. The ovaries were removed surgically and she takes estrogen replacement hormones.

Neuro exam shows yellow refractile bodies within the retinal artery. An outside physician does not alter her treatment or perform ancillary studies. These episodes persist and one month later she awakens with a left sided hemiparesis and hemianesthesia.


25. A 68 year old man with nonvalvular atrial fibrillation awakens one day and notices difficulty seeing on his right side; he has no difficulty with speech, walking, strength, sensation or coordination or headache.

The blood pressure is 110/80, pulse 80. The neurologic exam reveals a right homonymous hemianopsia. The heart is irregularly irregular and without murmurs. CT of the head is unremarkable.

The patient remains stable but a repeat CT three days later shows a hypodense left occipital lesion without mass effect and dense gyriform enhancement.


32. A 55 year old man undergoes 5 vessel coronary artery bypass grafting [CABG] for unstable angina. Prior to surgery he is found to have bilateral carotid bruits and duplex Doppler ultrasound shows 60% carotid stenosis belaterally. Brain CT and MRI show "bilateral periventricular abnormalities consistent with ischemic damage or demyelination." Following successful CABG, from which he makes a full and complete recovery, he undergoes bilateral carotid endarterectomy. Following this surgery, neurologic exam is nomral. He undergoes cardiac rehab and is pronounced fit to return to work in 6 weeks. At home, his wife initially supervises his bill paying, cooking and driving; she feels he is "okay." Three weeks after his return to work as a parmacist, he is terminated because he has made multiple "errors" and the patient has noted problems performing sequential tasks. His cardiologist evaluates him and on his mental state he has no abnormality, including 3 objects at 5 minutes, intact serial 7s and serial 3s. A psychiatric referral is suggested.

  1. What's the differential diagnosis?
  2. What other tests should be done?
  3. Could he have multi-infarct dementia?
  4. What is "pseudodementia?"

35. A 52 year old hypertensive, insulin-dependent diabetic man suddenly becomes mute and unable to express himself. His wife calls 911 and EMS arrives 10 minutes later. The patient can not express himself and has right hemiaresis (more in the arm than the leg). Despite the fact that glucose shows that he is not hypoglycemic, he is given 50 grams of glucose (an ampule of D50). He arrives at the hospital 20 minutes later. CT shows no abnormality. Recombinant tissue plasminogen activator (tPA) and he begins to speak and can move his right arm. Three days later, exam shows normal repetition and comprehension. Speech is nonfluent with low word output and agrammatical speech which is poorly articulated and without appropriate melody.

  1. What is wrong with this patient's speech?
  2. What has happened to this patient?
  3. Was there any potential danger in administering glucose to this patient?
  4. What might MRI show?

3. A 34 year old attorney wins a two million dollar judgment against a large HMO on Friday afternoon. After celebrating with champagne at Joe's that evening he experiences an episode of flashing lights beginning in his right visual field and moving slowly such that a "scotoma" follows this. Twenty minutes later the visual distrubance resolves and he develops left sided throbbing headache which builds up to maximal intensity over 3 hours. Activity makes the headache worse. He feels better lying down in a dark room. The past history is significant only for motion sickness as a child. His mother has hypertension and migraine headache. The neurologic exam is normal in the emergency room.


6. A 25 year old electrician falls off a ladder and strikes his head. He is unconscious for 20 minutes; when he regains consciousness he has amnesia for the event, but is otherwise normal. He develops a dischage from his nose and hematoma encircling his eyes. The next day he has a temporal headache and vertigo when he sits up. He goes to an emergency room, where a skull roentgenogram is normal.

He is afebrile. The neurologic exam reveals mild recent memory loss, anosmia, an ataxic gait. The neck is supple.

He is discharged from the ED with Antivert (meclizine) and Fiorinal (aspirin, caffeine and butalbital); two days later his headache persists and he is febrile. The neurologic exam is normal except for anosmia.


7. A 35 year old truck driver is involved in an MVA in which he strikes his head on the steering wheel. He is dazed and has a frontal bruise. He complains of headache and neck pain. Two days later, he reports an episode of feeling faint, light-headed and weak in his legs. He goes to the local Jiffy-Health Center (owned by a well-known HMO) and while being assessed by a nurse-receptionist he feels dizzy and passes out. The nurse observes right arm and left leg myoclonic jerks and he urinates on himself. Immediately after the episode, he is alert.

The neurologic exam is entirely normal except for accomodative paresis. The remainder of the physical reveals spasm of the trapezius muscle and limitation of neck movement laterally; flexion is supple.


30. A 25 year old man falls on a wet surface at Walgreens and lands on his buttock. He reports pain in the lower back and inability to walk straight, and demands to be taken to the Tulane ER. After a complete evaluation, he is found to be "normal" and referred to the neurology clinic. The following day he reports pain in his low back and entire left sided numbness and weakness. On neurologic exam

10.                         Mental status is normal

11.             Gait -- dragging of the left foot, which is held stiffly

12.             Motor -- inability to squeeze examiner's finger or extend the left wrist; inability to dorsiflex or plantar flex the left foot.

13.             Reflexes are 2+ and symmetrical; plantar flexion to sole stimulation bilaterally

14.             Sensory -- absent pinprick and light touch on left; absent proprioception and vibration at all joints on the left side

15.             CN -- reduced heading in the left ear and colors appear less bright in the left eye

Questions

16. Where's the lesion?

17.             What is the mechanism?

18.             What tests should be done?

19.             Define

1.              conversion disorder

2.                  Briquet syndrome

3.                  somatoform disorder

4.                hysteria

20.             Could this patient have multiple sclerosis?



31. A 7 year old boy falls off a swing and hit his head on a rock and is knocked unconscious for ten minutes. Upon awakening he is neurologically intact but does not remember the accident. Several hours later, he develops a headache, vomits and develops right sided weakness.

On exam BP 180/110 HR 50, RR 28

21. Mental status -- Lethargic but responsive

22.             Motor -- Right hemiparesis

23.             Reflexes bilateral plantar extensor response

24.             Cranial nerves -- left pupil is 8mm and poor reactive to light

25.             Fundi do NOT show spontaneous venous pulsations.

1.                 concussion

2.                 diffuse axonal injury

3.                 lucid interval

4.                 transtentorial herniation

5.                 tonsillar herniation

6.                 whiplash injury

7.                 lumbago

8.                 sciatica




36. A 42 year old man is evaluated for back pain. This is an aching pain which radiates to his foot. he is uncomfortable walking and can not find a comfortable position in bed. He consults his primary care provider and is treated with pain medication, muscle relaxants and an NSAID. He is no better; percocet proves ineffective. He reports a tingling sensation over the top of his right foot and he has tripped on two occasions.

Neuro exam shows

9.     absent right ankle jerk

10.             weaknes of dorsiflexion and plantar flexion

11.             decreased pinprick over the toop of the foot

12.             Loss of lumbar lordosis

13.             paraspinal muscle spasm in the lumbar region

14.             (+) Straight leg raise at 30 degrees on the right

15.             point tenderness over five lumbar vertebrae

16.             What structures are involved?

17.             What are potention mechanisms?

18.             What diagnostic studies are indicated?

19.             What are the clinical features of

1.              Brown-Sequard syndrome

2.                  transverse [thoracic] myelitis

3.                  anterior spinal artery ischemia

4.                conus medularis symdrome

5.                   cauda equina syndrome?

6.                  cervica syringomyelia



27. "Hurricane" Bill is mugged outside a bar in the French Quarter. He is unconscious for 15 minutes and has both anterograde and retrograde amnesia. The neurologic exam is normal except for abnormal olfaction. Ancillary studies include a skill roentgenogram which shows a linear parietal skull fraction. CT and MRI are normal.

Six months later he experiences an episode of lost awareness. This begins with funny sensation in his stomach which then rises to his chest. He then smells "burning rubber," and his wife states he has rhythmical blinking. After three minutes he is confused and complains of a left temporal headache.

20.                                    A. Has the patient suffered a concussion or a contusion?


22. A 23 year old injection drug user develops myalgia, fever and bifrontal headache. One week ago he had a 3 day episode of diarrhea, which responded to symptomatic therapy and resolved spontaneously.

On exam BP 140/80 HR 110 temperature 38.4C, respirations 12. The neck is supple and the skin is without rashes. The muscles are diffusely tender. The neurologic exam is normal.

WBC=10,000, 60% lymph, 40% PMN. ESR=40, U/A and chest film are normal.

A resident of a different speciality (and who tends to start patient descriptions with the phrase "this is the guy") believes "bacterial meningitis" or subarachnoid hemorrhage are likely possibilities (he does not consult neurology, a service he regularly belittles) and wants to ensure a nontraumatic tap. Since he read that the spinal interspaces are bigger higher up in the cord, he performs the tap at T9-10 instead of L3-4. Following this LP based on RANDO criteria (Resident Ain't Never Done One) the patient develops these imprairments:

24.                                    paraplegia with hypotonia

The ER staff asks you to answer the following questions so he can brief risk management

29.            A. Explain the post LP neurologic findings


11. A 35 year old electrician is involved in an MVA where he is struck from behind. Mild low back pain resolves completely after two days. Four weeks later he begins to trip frequently, and three days after that he has difficulty climbing stairs and opening jars.

The neurologic exam reveals

34.            weakness of the feet, hips, legs, knees and hands.

The physical exam is otherwise normal.

Impairment of which structures explains these findings?

38.            What diagnostic tests should be done?


17. A 30 year old hockey player is checked into the boards hard by the Flyers' Eric Lindros, striking his head and back, and briefly losing consciousness. (The rabid Philadelphia crowd goes wild.) He complains of occipital pain which is mild but also severe mid-thoracic back pain. Team physicians send him to the ER for observation. The following day the occipital pain is gone but the back pain persists and he reports a band of numbness on the left chest region extending to his back. He experiences difficulty urinating.

Neurologic exam reveals mild weakness in both legs; the arms are normal. Reflexes at the ankle,knee and abdominals are present but depressed, and there are plantar responses bilaterally. There is decreased perception of pinprick from the umbilicus downward, and there is a band of decreased seneation on the left mid thoracic and back region. There is a reduced anal wink.

43.                                    What was the mechanism of the initial injury?


4. A 70 year old judge notes deterioration in his handwriting such that clerks can not read it. (See sample.) He also has difficulty cutting his food and bringing a cup to his mouth. He jokes that has always been shaky and that this runs in his family.

Neurologic exam reveals normal gait and station. There is a motor sustention and intention arm tremor; mild tremor of his voice but no rigidity or bradykinesia; Myerson's sign (-). A drawing of Archimedes spiral is presented.

47.                                    What is the differential diagnosis?

.


15. A 15 year old adolescent is described as being "nervous and fidgety" (more than appropriate for age, all things considered). His performance in school and his behavior become bizarre. Past history is significant for viral hepatitis 2 years ago and unexplained hemolytic anemia 4 years ago. Both his parents are diagnosed with schizophrenia.

On exam he is inattentive with motor restlestness. The gait is broad based with impaired postural reflexes. Motor exam reveals cogwheel rigidity and intentional tremor. Finger-to-nose and heel-to-shin testing are impaired. The cranial nerves and fundi are normal. The physical exam reveals hepatosplenomegaly; the skin is normal.

53.                                    What are the diagnositic possibilities?


16. A 70 year old physician is brought to a neurologist by his family because of "failing memory." He forgets to pay his bills, has lost his house keys and gotten lost while driving. Recently, three patients have initiated malpractice claims due to his failure to diagnose their condition correctly. He has become depressed and recently locked himself in the bathrom. He appears to move slowly and has suffered multiple recent falls.

Neurologic exam reveals a Mini-Mental Status Exam of 19/30. The gait is slow and he has difficulty initiating movement. Postural stability is impaired. Strength is normal and coordinated movements are slow. There is bilateral cogwheel rigidity but no tremor.

Gaze both up and down is impaired. Myerson's sign is present (+) and there is neck rigidity.

57.                                    What neurological impairments does this patient suffer from?


20.A 40 year old injection drug user, while walking down the street minding his own business, is shot in the left orbit and the bullet damages the left frontal brain region. He undergoes bifrontal craniotomy for bullet removal and brain debridement. Following surgery, he is alert and attentive with no aphasia or memory impairment. He has no motor coordination, gait, hearing or visual disturbance (other than left eye blindness). He makes an uneventful recovery but his wife calls the neurosurgeon very upset. He refuses to return to work or pay any bills, goes to bars and Harrah's Casino and loses large sums of money, refuses to go to church and sexually abuses a 6 year old girl and is arrested for indecent exposure.

Assuming this behavior is a dramatic step down from his life injecting drugs

62.            Explain the possible mechanism of this behavior which his wife says is a dramatic change


19. A 25 year old pregnant woman goes into active labor. She has a generalized seizure due to eclampsia. She is treated with fosphenytoin and magnesium sulfate and has no further seizures. During labor she receives generous amounts of fluid due to possible dehydration. Four hours later she is delivers a healthy baby girl by spontaneous vaginal delivery. In the recovery room she complains of generalized weakness which progressively worsens. Examination shows both proximal and distal weakness in her arms and legs, normal sensation and absent reflexes, normal cranial nerves. Over the next 24 hours this weakness progresses to quadriplegia with respiratory disturbances.

65.            What are potential mechanisms of the quadriplegia?


21. An 18 year old man (HM) had a fall from his bicycle at age 6 and struck his head. He was unconscious for several hours. Skull roentgenogram showed a left parietal linear skull fracture. Six months later he had "staring spells" preceded by olfactory aura. Three months later he had generalized seizure. Despite treatment with multiple medications seizures occur on a daily basis. EEG shows bitemporal slow wave activity and right temporal spikes. MRI shows right temporal lobe atrophy. He undergoes right temporal lobectomy for seizure control. Following surgery he has no further seizures, but reports a problem with his memory. When the patient has breakfast he can not remember eating 15 minutes later. He talks to the examiner, but cannot remember the conversation 20 minutes later. His memory is generally impaired for recent but not remote events.

69.            What has happened?


12. A 25 year old woman slips and falls in a shopping mall on a soap sample outside the Body Shop. She complains of low back pain which does not radiate down her legs. (Also, the soap smells terrible.) She calls her physician who calls in a prescription for an NSAID and a muscle relaxant. Five days later she continues to have severe pain and demands to see a neurologist.

Neurologic exam reveals a broad based gait with difficulty standing on her heels. Romberg's sign is present. There is weakness of dorsiflexion and evertion of the feet. There is decreased vibration and position sense in the legs. Reflexes are absent in the ankles. The cranial nerves are normal. The general exam reveals high arched feet (of which the patient has always been proud), and hammer toes and pes cavus deformity (of which she has not), paraspinal muscle spasm, sciatic notch tenderness. MRI of the lumbar spine shows L4-5 and L5-S1 disc protrusion.

72.                                    What is the relationship of the fall to the neurologic findings?

.


23. A 37 year old man develops polydipsia and polyuria. Lab studies confirm the diagnosis of diabetes mellitus and treatment with oral hypoglycemic agents is started. Two months later he complains of an uncomfortable burning sensation in the soles of his feet, worse at night and not present when he walks. The neurologic exam reveals absent vibratory sense in the toes and reduced at the ankles, reduced pinprick and temperature sensation in the feet, absent ankle reflexes, and absent response to stimulation of the sole and a normal motor exam.

76.            These clinical findings are best explained by what abnormality?


28. An 18 year old has been weak since childhod, has never been able to walk normally and has been convined to a wheelchair for 2 years.As a child he could not run or climb stairs or arise from a chair without using his hands to push off. A maternal uncle died at age 20 of progressive neurological illness, but the parents are normal. He has no siblings.

The neurologic exam reveals scoliosis, winging of the scapula, flexion contractures of the ankles, knees and hips. There is marked muscle weakness in the arms and legs (proximal greater than distal) and marked wasted, except for the calves, where bulk is preserved. Reflexes are present only in the ankles and triceps. The cranial nerves are normal.

CPK=3,000. EMG reveals myopathic potentials. Nerve conduction velocities are normal. A muscle biopsy reveals muscle atrophy with fat and conective tissue replacing muscle; dystrophin is shown to be absent by special staining.

82.                                    What type of myopathy does the patient have?


29. Harriet is an 18 year college freshman who develops abdominal pain and vomiting followed by diarrhea the next day, and a dry mouth. One day later she develops droopy eyelids, diplopia and difficulty standing and difficulty lifting heavy objects.

The neurologic exam reveals ptosis, bilateral facial weakness, pupils are 5mm and poorly reactive, reflexes are absent, and proximal greater then distal weakness. The sensory exam is normal. A chest film reveals a left lower lobe pneumonia; an abdominal film reveals colonic distention.

85.                                    What neuro impairments might cause these findings?


10. A 33 year old woman complains of blurred vision in her left eye and pain in the left orbital region (as if she had sand in her eye) especially when she moves her eyes. She has no headache, double vision, facial parasthesieas, or numbness. She has recently felt fatigued enough that she has started working part-time rather than full-time, has had multiple urinary tract infections and urinary frequency with nocturia. She complains that her legs become stiff when she walks long distances.

Visual acuity in the left eye is 20/70, in the right eye 20/20. The fundi are normal. There is a left centro-cecal scotoma. The left pupil does not react when light is shone into it; when light is shone into the right pupil both pupils contract, and when you swing the light in front of the left it dilates (afferent pupillary defect). The patient is generally hyperreflexic with plantar responses. Abdominal reflexes are intact and present; sensory and cerebellar exam are normal.

93.                                    Where's the lesion?


5. A 50 year old alcoholic stops drinking due to severe abdominal pain. That day he has a generalized seizure and is admitted to the hospital. He is confused, ataxic and reports double vision. His T-shirt reads "Rehab is for Quitters." Treatment with Ativan (lorazepam), Librium (chlordiazepoxide) and phenobarbital is instituted. He becomes increasingly lethargic and then obtunded.

The physical exam reveals icteric sclerae, multiple spider angiomas and an enlarged and tender liver. The neurologic exam reveals a lethargic mental status. There is *asterixis* no pronator drift and normal strength. He is generally hyperreflexic with bilateral extensor plantar responses. The sensory exam is normal. Cranial nerve exam reveals horizontal and vertical nystagmus and a left lateral rectus palsy. The fundi are normal.

Pertinent laboratory data (from the chem everything sent by the ER) includes an elevated bilirubin and alkaline phosphatase, a normal blood glucose, WBC 6000 and Hgb 12.1.

Based on the clinical history and exam findings:

98.            Explain the mechanism of the neurological and medical disorder.


13. A 22 year old man whose roommate has recently died of AIDS becomes very depresed and suicidal. He injects himself with a blood of another HIV(+) friend so that they can "bond." Four weeks later he develops sore throat, fever, arthralgias and lymph node swellings. He then develops facial weakness, neck muscle weakness and shoulder weakness.

The neurologic exam reveals weakness of the entire face bilaterally, neck and proximal arm weakness, absent biceps and triceps reflexes. The CSF is acellular and has an elevated protein. Nerve conduction velocities are markedly slowed and electromyography shows normal muscle potentials.

100.                                What has happened to this patient?

After complete recovery he reports that his memory is poor and his thinking is slowed. He can not concentrate or sustain attention. He has difficulty sleeping and poor appetite. Neurologic exam reveals a Mini Mental Status Exam score of 27/30, but is otherwise normal.

102.        What is wrong with this patient?


14. A 28 year old male with polysubstance abuse (cocaine, heroin, PCP) develops headache and muscle pain. He reports night sweats and loss of appetite due to difficulty swallowing. Two days later he reports back, abdominal and jaw stiffness. The past history is relevant for bacterial endocarditis treated with IV antibiotics 3 years ago and chronic schizophrenia treated with stelazine, haloperidol and clozapine, but the patient is usually non-compliant with these medications.

BP 200/100, HR 120, RR 22, temp 101. The neurologic exam reveals an alert, attentive man. His gait is stiff and antalgesic. Truncal rigidity is present but motor strength is normal. The reflexes are brisk and symmetrical, with bilateral plantar responses. Sensation is normal. The cranial nerves reveals orofacial dyskinesias with facial and jaw spasms and nuchal rigidity. There are multiple wounds. Laboratory studies reveals WBC 18,000 with a left shift, normal thyroid studies, CPK 2000, urine negative for myoglobin. A CT done by the ER is normal, and an LP done in the ER on RANDO criteria (Resident Ain't Never Done One) is normal as well. ECG reveals sinus tachycardia only. An EEG is normal.

The ER initially consults psychiatry for effect of antipsychotics, but he goes to medicine to be "medically cleared." Neurology is consulted when the patient becomes confused and develops generalized myoclonic jerks (described to you as seizure activity by the medicine resident). Respiratory stridor develops, the blood pressure rises to 220/130 and cardiac arrythmias develop.

105.                                What is the differential, and what is the diagnosis?


8. A 26 year old obese woman develops headache, double vision and a discharge from her left breast. Her last menstrual cycle occurred one year ago. She feels tired all the time and reports that her hair is falling out. The neurologic exam reveals a left lateral rectus paresis, decreased sensation on the left side of the uper face. The visual fields are full and the fundi are normal with spontaneous venous pulsations. The pupils are equal in size and react normally to accomodation and light.

108.        Where's the lesion?


9. Six months following trans-sphenoidal hypophysectomy for a prolactin-secreting macroadenoma, headache and double vision recur. The headache is bi-frontal, episodic, increases with cough or sneezing and awakens her from sleep. Diplopia is horizontal and is maximal on far gaze. Neurologic exam now reveals a right lateral rectus paresis; the fundi are without spontaneous venous pulsations and there is blurring of the disc margins with centrally located hemorrhages and exudates. Visual acuity is 20/50 in the left eye with reduced perception of color in the left eye; the right eye is normal.

112.        What is the most likely explanation?


26. A 32 year old woman has difficulty understanding what people are saying in telephone conversations and in noisy crowded rooms. This is a major problem when she holds the phone to the right ear. She also notes intermittent slurring of speech and right facial numbness. The neurologic exam reveals decreased hearing in the right ear, mild right nasolabial flattening, an unsteady tandem gait, a decreased corneal reflex on the right. Using a 512Hz tuning fork, hearing is depressed on the right and air conduction persists when bone conduction terminates. Audiology reveals a high frequency loss on the right and impaired speech discrimination, and decreased brain stem auditory evoked potential on the right. Response to caloric testing on the right is decreased.

118.        What conditions should be considered?


37. A 52 year old legal secretary complains of right wrist pain. This is worse with activity and recurs at night. She has difficulty writing and using tools, and says she can not work due to tingling numbness in the fingers. Neurologic exam shows:

124.        weakness of thumb flexion

Questions

129.        What neurological condition explains the findings?

While driving to work she is struck by the car behind her. Her head hits the steering wheel. The next day she has neck stiffness and soreness. Physical exam shows:

133.        Neck exam shows straightening of the cervical lordosis and cervical paraspinal muscle spasm


38. A 10 year old boy falls on the ice and breaks his left radius and ulna. He is treated with a cast for 6 weeks. Following removal of the cast, he reports difficulty opening jars and cans.

Neurologic exam shows

141.        weakness of wrist flexion and finger adduction


39. A 40 year old homeless alcoholic sleeps on a park bench. When he awakens he has no use of his right hand. He has no headache, visual symptoms, numbness or difficulty walking.

Neurologic exam shows:

146.        weakness of wrist extension


40. An 18 year old Tulane co-ed leaves for Florida by car on spring break. One week previously she started on an oral contraceptive pill. While sleeping in the back seat her friends report that she has frequent muscle jerks. In Florida she has generalized seizure activity after drinking a six pack of Miller Lite, and is dazed and confused for the next two hours.

On physical exam

153.        temp 98.4F, BP 110/80, HR 70 and RR a suspiciously normal 20

Lab data

157.        WBC 10, 75% segs, 15% bands, 10% lymphs


41. The student from the previous question refuses to take any anti-epileptic drug. She has no futher seizures until the day of last final exam, which occurs after she stays up all night with the help of No-Doz, Ritalin (given to her by a friend diagnosed with ADHD) and lots of coffee.

Physical and neurologic exams are normal.

She is started on carbamazepine (Tegretol), 200 mg TID. She compliant with treatment and therapeutic blood levels. Eight months later she has an episode during which she feels dizzy, light headed, weak, falls to the ground and is briefly unconscious; there is no post-ictal confusion.

167.                                What is the likely mechanism?


33. An 18 year old college freshman is found apneic and pulseless in her boyfriend's room after attending a fraternity party after the Tulane-LSU football game. CPR is initiated by her boyfriend; pulse and blood pressure are noted when paramedics arrive but she requires assisted ventilation. She is stabilized.

Exam reveals

170.        Coma

The next day respirations are spontaneous, pupils are reactive but she remains comatose. One week later her eyes are open but she shows no response to verbal stimulus. Eyes rove spontaneously but she does not follow objects in her environment and she has no reponse to auditory stimulus. She has limb movement to noxious stimulus but no voluntary movements; she continues to require ventilatory support. CT and MRI are normal, EEG shows low voltage theta waves.

175.        What happened to her?


34. A 35 year old supervisor of a nuclear energy plant begins to have episodes of falling asleep on the job. These occur at work and while driving. (Feel free to hum the theme from 'the Simpsons' now.) He has multiple near miss accidents and begins to ahve his friends drive him. After the episodes of excessive daytime sleepiness (each lasting 5 to 10 minutes)he feels better, more alert and refreshed. He is very concerned because these attacks are increasing in frequency and now occur multiple times daily. Also, he reports that when he laughts he feels weak an doccasionally falls to the ground. Family history is relevant that his mother and brother have epilepsy. Neurologic exam is normal.

Questions

177.        What's the diagnosis?


42. A 28 year old man slips and falls on his buttock. He complains of low back pain which persists despite bed rest and treatment with naproxen (an NSAID), carisoprodol (Soma, a muscle relaxant), and tramadol (Ultram, a serotonergic pain reliever).

Physical exam

182.        Normal lumbar lordosis

The pain persist and interferes with sleep. Amitriptyline is started in addition to other medications. One week later he has a generalized seizure. At this time, neurologic exam and EEG are normal.

187.        What is the likely mechanism of the seizure?


43. Sally is a 35 year old woman who has a history of sinus headache. She has strongly positive family history of headache. She had motion sickness as a child. At menarche, she develops episodic left temporal throbbing headache associated with nausea, photophobia and worsened by activity. Her headache responds to sumatriptan and occurs once per month. One decade later, she develops chronic daily heacache which is aching in type, associated with neck pain. She becomes increasingly depressed and anxious.

One night after 6 days of headache severe enough cause her to miss work, she goes to the ER with her bag of 23 different medications including hydromorphone (Dilaudid) and codeine/aspirin (Percodan) for a second opinion (yours).

190.        Classify her headache pattern.


44. A 20 year old college student has an accident when an ice sculpture strikes her in the left frontal region. She has no loss of consciousness or amnesia for the event. She has mild headache and heck pain which resolves spontaneously. Three months later she has an episode of right facial twitching followed by generalized tonic-clonic activity, incontinence and post-ictal confusion.

Neurologic exam shows

193.        a R pronator drift


45. There's a 25 year old nurse in the ER nobody likes very much. We'll call her Mary. (Actually, everybody calls her 'that bitch,' but we'll be nice and call her Mary too.) One day she develops a severe occipital headache after an altercation with a colleague. The pain is intense and very uncomfortable, and after trying Aspirin and Tylenol without relief she goes to you for help. She has no prior headache history, and no history of headache in her family.

The neurologic exam is normal, as is a computed tomogram of the head.

200.                                What is the diffential diagnosis?


46. Randy is HIV positive and takes highly active anti-retroviral therapy. He has a normal CD4 count and an undetectable viral load. He has no fatigue or weight loss. He reports new onset heacache, bitemporally, throbbing in nature, building up over 6 hours and ssociated with nausea.

On exam

204.        Neurologic exam is normal

What is the explanation for these findings?

210.        What are alternative explanations?


47. Sam is a 67 year old retired CPA. He develops left temporal aching pain and visual blurring. He has myalgias and pain on chewing food.

Neurologic exam shows

211.        left temporal artery tenderness


48. Immediately following delivery of her third child, Mary develops severe headache, diplopia and left eye blurring.

On exam

219.        BP 95/50, taken by you with a manual cuff and HR 110


49. harry is a 52 year old math teacher. He has hypertension and hypercholesteremia. Medications include Accupril (an ACE inhibitor) and pravastatin (Pravachol). One morning he awakens with L eye visual blurring as if a shade has been pulled down; he closes the L eye and his vision is normal in about 15 minutes. He then develops left sided aching neck pain which resolves with over-the-counter analgesics. He calls his physician, who makes a diagnosis of "ocular migraine" over the phone and tells him to make a routine appointment in 10 days for a checkup. His wife is not pleased with this advice and insists he see a neurologist (i.e. you).

On exam

230.        bilateral carotid bruits


50. A 23 year old woman develops episodic headache lasting 30 minutes associated with diplopia on far gaze while driving. She has 6 episodes daily and they do not respond to medication. Other medications include tetracyclinc and tri-orthocycline for acne. She has gained 40 pounds in the past year.

Neurologic exam shows

239.        absence of venous pulsations and disc elevation in the eye


51. An 8 year old boy develops intermittent headache and falls when running. He has intermittent horizontal diplopia when watching TV.

Neurologic exam shows

247.        broad based ataxic gait, falling to left


1a. A 26 year old female graduate student was conducting a philosophy seminar when she suddenly started stuttering and then became incoherent. She seemed confused, and her mouth was twisted. One arm hung limply and she walked unsteadily. She had a past history of theumatic heart disease and took no meds except for birth control pills.

256.        What type of neurologic problem is this?


2a. A 68 year white female presents complaining of inability to walk. Upon further questioning you find that this has progressed over a month or two and is not associated with back pain. On exam she is slightly inattentive and sometiems inappropriate. Language is intact. She has no cranail nerve deficits, and good strength in the upper extremities. Her legs are diffusely weak, 3-4 of 5, proximally and distally. Sensory exam reveals questionable mild loss of light touch and pinprick distally over the legs, without a level. Reflexes are brisk in the legs and a Babinski sign is present bilaterally.

260.        Where's the lesion?

.


3a. A 55 year old black female with a history of diabetes and hypertension states that while drinking her morning coffee she suddenly experienced "heviness" of the right arm. She fumbled with her cup until she spilled the coffee, and when the symptoms did not resolve within half an hour she came to the ER. Examination reveals an alert woman with normal mental status, and decreased light touch, pinprick, vibratory sense over the right arm and leg. Strength is normal.

262.        Where's the lesion?

 


4a. While working out at Reily uptown with free weights, your friend complains of sudden back pain and inability to walk. Your exam shows bilateral leg weakness, absent ankle reflexes (assume you have a reflex hammer in your gym bag), and decreased tone in the legs. He feels parasthesias running down the back of both legs, and doesn't notice you sticking his skin with a clean pin until the mid-thigh. Pressure on the lumbar spine is painful, and there is paralumbar muscle spasm.

266.        What happened?

 


5a. A 35 year old black male is seen in clinic with a 3 month history of weakness and muscle cramps, first felt in the left arm but progressing to both legs. His voice is not as loud as it used to be, and is a little horse. Sometimes food gets "stuck" on one side of his mouth and he has to move it with his finger. He has no sensory loss. Reflexes are brisk, including a jaw jerk. The toes are equivocal. Fasciculations are present in the tongue at rest, and in all four proximal extremities. He is lost to follow up, but returns a year later complaining of trouble swallowing, shortness of breath and appears emaciated.

269.        Where's the lesion?

.


6a. A 62 year old woman complains of painand numbness of the hand. She has been dropping objects from the hand, but the discomfort is worse at night.

273.        What further history would help you localize the lesion?

On examination you also find a loss of pinprick at the toes, and vibrations with a 128 Hz fork are felt for 10-5 seconds at the toes.

275.        What do you suspect now?

 


7a. A 26 year old woman is referred to you by her psychiatrist. For the last year she has complained of weakness which came on after the death of her father. The weakness seems to come and go depending on her family situation and her depression. She also complains of a vague tightness of throat (?globus hystericus), leg aching and frequent headaches. Sometimes she is fine, and other times she just lays on the couch, or will suddenly fall walking off a curb. She admits she doesn't know how to "pull herself out of this."

277.        How will you proceed?

.


8a. A 30 year old white female has difficulty climbing stairs. She cannot lift objects but has no problems writing or buttoning her shirt. Her gait is waddling. She has been followed for 8 months in rheumatology clinic for "arthritis" not otherwise specified, and a visit to the walk-in clinic prompted her appointment with neurology. The joints seem normal to your exam, but the thigh muscles are tender to palpation?

 

280.                                Where's the lesion?

.


9a. A 28 year old white female complains of headaches for one year, recently daily. They are often throbbing, usually bitemporal and do not usually cause too much nausea, although she has vomited once or twice. She also says her vision has changed, but she went to get her glasses checked and was told they were fine. Other pertinent history is obtained that she had a child 9 months ago and gained 80 pounds during the pregnancy, of which she has lost 30.

On physical exam, she is obese. Vital signs are normal. Funduscopic exam shows bilateral disc margin blurring with a flame hemorrhage on the right. Pupils are equally reactive, and visual fields are full to confrontation. There is a question of mild lateral rectus weakness on the right. The remainer of the cranial nerve exam is normal, as is stength, sensation and reflexes, and gait.

CT of the head is normal, as is an EEG. On lumbar puncture, the opening pressure is 41 cm H20.

282.                                What is this syndrome?